Вы находитесь на странице: 1из 93

T C S

P r e v i o u s

Q u e s t i o n s

A t t a c k

T C S

FYI (For Your Info)


TCS Test Pattern:
It was an online exam. Test Consists of 3 Sections
1.

Test on Synonyms and Antonyms (40 questions, 20 minutes: Also Qs on Sentence Completion).

2.

Aptitude Test (32 questions, 40 minutes: General Questions on Mathematics).

3.

Critical Reasoning (30 Minutes: 3 Analysis Paragraphs and 12 questions Relating them).

4.

Psychometric Test (150 Questions, 30 minutes: only if you Qualify in the above tests).

Details about How to Prepare for the above tests:


1.

Verbal Ability:
This Test contains Synonyms & AntonymsAlmost all of them were From GRE BARRONS 12 th Edition &

upward30-50% of them will be repeated from previous TCS papers. Few Questions on Sentence Completion were also
there..for these also refer BARRONS
NOTE:
During the Exam you should be careful because on the computer screen the names Synonyms & Antonyms will be
placed side by side so that we may by mistake choose an antonym instead of a synonymso Be Careful..
2.

Aptitude Test:

This test will consist of very general questionsThis is the easiest section Compared to the Other two. Almost 70-80
Questions will be repeated from the Previous TCS Papersjust learn the method from those papers same model will be
given..(This happen almost all the times). If you have time refer to any Aptitude book of your interestit may be
R.S.AGARWAL or any Other
3.

Critical Reasoning:

Here Starts your Problems30 Minutes12 questions..dont think that there is a lot of Time..these will be somewhat tougher
those who has some touch with such kind..and will be Very Tough.. if you havent seen this kind before..so,you should have
some practice relating this kind of question
3 paragraphs will be given.. 3X4.. 12 questions.. first 2 paragraphs will be some twisty questions like our
arrangements questions routes between cities.. like that and the last one will be on Data Interpretation

you

prepare Data Interpretation again from BARRONS. but the remaining from R.S.AGARWAL-Reasoning. Or from Barrons
13th Edition
Psychometric Test:
This test comes up only if you qualify in the Aptitude test This test consists of 150 questions which we should
answer in 30 minutes (of course, that much time is not required) Dont think that this test has no weightage.. there is
filtering in this test too this test tests your psychology.. Remember among 150 many questions were asked again & again
in a variety of ways.. so dont get trapped be honest while attempting this test.. choose what youre your heart says.. it will
take you through the test..
This ends the Tests
INTERVIEWS:
Now if you are Alive Even after All these tests.then you have to face a Technical Interview It may be on the same day of
the Test.. so be Prepared in Advance.. The Interview will be done by a Panel of 2 members. No need to worry.. They will
be Very supporting & Friendly.. First they ask us to Introduce ourselves then theyll go into your project details dont
forget to prepare project. They mainly concentrate on your project onlyso be confident on every aspect of your project
then you should have some basic knowledge on C language. There will be some questions like what is the o/p of this code?,
what are the errors in the code?..like that so be prepared for all the basic concepts.. If needed you have to write some
small codes too.. at the end they will ask some general questios like Why TCS?, Where do you see yourself in the next five
years?, do you have any questions?, try ask some questions it shows your interest in the Company..
HR:
Now if Chase this one too. Then comes your Final Attack.. the HR Round I many times heard that I failed in HR round!..
likewise anwers. But you should not fail in it.. because it involves no technical questions it just dwell into your innerself
Edited by: Riyaz Ali Khan Mayana

B. Tech .(CSE), D. E. C. E.

Email: riyaz666@gmail.com

so, be confident and frank while facing this roundThey generally ask questions like Tell about yourself?, Who r your role
models?, What is your weakness?, dont say I Have no weakness!, just prepare a small weakness for
yourself that causes no harm to your futureO.K.. What is your mother tongue? if your Academic background is good.. be
prepared to face some Techies here too.
NOW IF YOU ARE SUCCESSFUL HERE TOO.. CONGRATS YOU ARE IN TCS..

Contents
1.

Verbal

2.

Quantitative

3.

C- Aptitude

30

4.

Psychometric

61
VERBAL

i am sending almost all the words that r coming the papers.the meanings r also provided.as far as i am concerned all the
meanings r correct but if u have any doubts,verify or cross check.they come in synonym part of antonym part.i dont know
what will come for what.but get the meaning u can Ans.wer it .jst get them by heart,u will get through the vocabulary.but
bear in mind ,tcs has an upper cutoff.so even if u know all the bits in the vocabulary,dont Ans.wer all the 20.at the max
Ans.wer 18.
Synonyms:
merry = gay

relinquish=to give up,renounce

alienate= estrange

trAns.ient=momentary,not lasting

solicit = beechat,urge,humble

parley=conference

heap= pile

vitate=not in operation

cargo= freight

hilariry=comical

momentary= trAns.ient

expound=to expose,to explain

volume= quantity

sinister=evil

veer = diverge

retrogate=to go back

dispel= dissipate

salient=prominent

admonish= cautious,usurp

nascent=emerging,budding

meager= scanty

exonerate=freeing from burden,go away

latitude=scope

appal=shock

latent= potential

anomaly=irregularity

covet=crave,to desire

embrace=to take in hands

discretion=prudence

acumen=shrewdness

emancipate=liberate

ordain=command

lethargy=stupor

florid=ornate

concur=acquiesce

penitence=compunction

confiscate=appropriate

whet=stimulate

baffle=frustrate

mortify=humiliate

subside=wane

adage=proveb

misery=distress

rapt=concealed

pretentious=ostentatious

cajole=coax

tranquil=serene,silent
Edited by: Riyaz Ali Khan Mayana

abode=dwellling
B. Tech .(CSE), D. E. C. E.

Email: riyaz666@gmail.com

efface=obliterate

extricate=isolate,liberate

obstinate=stubborn

pious=devout

hover=linger

ovation=applause

caprice=whim

incentive=provacation

belate=too late

indignity=unworthiness,disgrace

renounce=reject

inept=unfit

brim=border

infirmity=feeble,weak

divulge=reveal

candid=frank

lament=wail

dangle=to hang loosely or with a swinging motion

bileaf=conviction

restiveness=unwillling to go forward

adhesive=tenacious,sticky

irksome=causing uneasiness

furtive=stealthy

jaunty=having an airy mannered approach

hamper=obstruct

nebuluos=misty,hazy,vague

to merit=to deserve

decollete=with neck uncovered

inert= passive

misapprehension=understanding in a wrong manner

stiffle=sniths

obloquy=reproachful language,disgrace

instigate=incite

censure=criticize,approve

depreciation=deflation,fall in value

optimum=favourable

deprecate=express disapproval

cite=quote

echelon=level of authority

effusive=overenthusiastic,spontaneous

intermittent=externally stopping and starting

irradiate=radiate

detrimental=harmful

tenacious=firm,stubborn

abberation=deviation

voluble=talkative,loquacious

conciliation =to make friendly

banal=commonplace,trite

orthodox=conventional,superstituous

generic=general

fallable=liable to err

emperical=mathematical

volatile=ever changing

circuitous=roundabout,indirect

manifestation=clear or obvious

surveillance=observation

connotation=suggest in addition to fundamental meaning

raucous=harsh,rasping

circumspect=cautious,prudent

voracious=greedy,hungry

absymal=bottomless,unclear

pedigree=ancestry,derivation

illustrious=highly distinguished,nodal

fidelity=loyalty,commitment

prolific=abundantly fruitful

augment=add to ,magnify

delight=high pleasure

precarious=unstable,uncertain,hazardous

vehement=violent,forceful,urgent

alacrity=cheerful promptness

impetus=moving force,stimulus

derogatory=detracting,injurious

acronym=a word formed

onus=responsibility

disseminate=scatter

analogous=comparable

harbinger=forerunner,announcer

expedient=suitable,advisable

tractable=docile,manageable

assuage=to soften,to mitigate

covert=secret,hidden,implied

compliance=yielding,agreement

pensive-sadness,contemplate

diffidant=shy

mitigate=to appease,to moderate

plaintive=lamenting

clutch=to hold

insinuate=hint,intimate

motley=multicoloured

misdeamoner=bad conduct

attenuate=to weaken

gregarious=helpful

sonorous=resonant

anathema=curse

bolster=support

benign=kind

divergent=deviating

recapitulate=review

decollate=behead

slump=to fall in mud

Edited by: Riyaz Ali Khan Mayana

B. Tech .(CSE), D. E. C. E.

Email: riyaz666@gmail.com

heterodox=having a different opinion

trAns.lucent=partially trAns.parent

ignonimous=dishonour,disgrace

sagacity=keen in perception or thought

effigy = dummy

cacophony=a disagreeable sound

sacrosanct=too important

jaded=a sorry horse,illnatured woman

cryptic=hidden,secret

mien=look,manner

debilitate=to weaken

buxom=gay,jolly

sceptic=doubt

foray= to raid

erroneous=full of error

denoument=final outcome of an event

minion=darling,flatterer,a sevile dependent

behest=hold down

veracity=truthfulness

erratic=unknown

brackish=somewhat saline

disparity=unequaity

cogent=powerful

ovulate=fertilize

taciturn=unspoken

whimsical=fanciful

liasion=union,bond,connection

dogmatic=arbitrary,opinionated

ramification=branching

daze= to stun

semblance=likeness,outward show

scathing=to injure,hurt

impasse=a place from where ther is no outlet

icon=figure,portrait

exasperate=to make rough,to irritate somebody to a high

fraught=cargo

degree

pithy=strong,energetic,

invoke= to call uppon

isotropic=having same properties

docile=teachable,

expidite=hasten

aegis=protection

adversary=opponent

exacting=compelling full payment,unreasonable demand

affable=lovable

fiasco=failure

agregious= apart from croud,extremely bad

intercept=to cut

conglomeration=group

arbitrary=not bound by rules

erudite=wise,profound

categorical=positive,absolute,without exception

augury=prediction

protract=to prolong,drawn to scale

constituent=accompanying

obliterate=to blot out,rub out

differential=having or showing or making use of

altercation=heated arguement

distention=act out,stretch out

dwindle=to grow less,to waste away

litigation=engaging in a law suite

efface=rub

preponderant=superiority of power

agrarian=related to agriculture

precipitate=quicken

vacillate=dillema

repugn=to fight against

experdent=fitting proper

repugnance=inconsistency

simulate=produce artificially

slur=thin mud

cognizance=knowledge

protean=variable,inconstant

retrospective=review

balmy=mild

naive=innocent

recalcitrant=stubborn

eqvivocate=tallying on both sides

guile=deceive

affinity=strong bond

ponderous=weighty,heavy

stilted=elevating

baleful=malignant,painful,hurtful

asperity=roughness,harshness

intrepid=brave

felicitious=happy,prosperous

evanescent=fleeting,fading away

bouillon=boiled meat

genuflect=bend down as respect

pillage= act of plundering

stoic=indifference to pleasure or pain

physiognamy= the act of judging one character from

hesitant=to stop making a decision

apperance

proclivity=inclination,prone

repudiate=to divorce,to reject

conjecture=a forecast,a guess,

inundate=to flow uponto overwhelm

mundane=lonely,earthly

Edited by: Riyaz Ali Khan Mayana

B. Tech .(CSE), D. E. C. E.

Email: riyaz666@gmail.com

bilk=to elude,to cheat

mollify=to soften,to abate

nettle=to sting with annnoyance

equanimity=evenness of mind

mulch=loose material

gist=main point of a matter

impugn=oppose

contraband=excluded by law

sobriety=habit of being sober

awry=distorted

misanthrope=hater of mankind

frugality=economy

waif=wanderer,ownerless property

repartee=ready and witty retort

debacle=breakup or collapse

boisterous=noisy

tarry=to linger,to delay

ungainly=awkard,clumsy

incontinent=not restraining the passions

profound=deep seated,intense

paradox=contrary to the received opinion

incorrigible=beyond correction

lackluster=dull

interdict=prohibit

moribund=dying

cohere=held together

callow=youthful

unreserved

Censure fault, criticize

Benign kind, benevolent, compassionate

Optimum best, most favorable

Attenuate satisfy, calm, soothe, ease

Candid frank, open, blunt, upfront, forth-right

Sonorous loud, deep, resonant, echoing

Cite quote, name, mention, refer to, allude to

Bolster boost, strengthen, reinforce, encourage

Effusive demonstrative, fussy, talkative,

Heterodox unorthodox, dissenting, contrary to accepted

overenthusiastic, vociferous, extroverted

belief, heretical, deviating

Voluble articulate, vociferous, talkative

Restiveness impatience, restlessness, nervousness

Banal commonplace, trivial, predictable, trite, hackneyed

Effigy image, statue, model

,unoriginal

Retrograde retrospective, traditional, conservative,

Standing rank, permanent, position, duration, status,

nostalgic,forward-looking(antonym)

reputation, eminence

Sacrosanct sacred, holy, revered

Nascent budding, emerging, blossoming, embryonic

Dangle hang down, sway, droop, swing, suspend

Clutch grasp, grab, hold

Cryptic mysterious, enigmatic, puzzling, hidden

Generic general, basic, common

Debilitate incapacitate, weaken, hamper, encumber, hinder

Empirical experimental, pragmatic, practical

Divulge reveal, disclose

Anomaly irregularity, glitch, difference

Spendthrift wastrel, squanderer, compulsive shopper

Circuitous roundabout, twisty, meandering, indirect,

Indigenous native, original, local

winding, tortuous

Erroneous mistaken, flawed, incorrect

Surveillance observation, watch, shadowing

Minion follower, subordinate, underling, gofer

Objective aim, impartial, real, purpose, goal

Veracity reality, truth, sincerity

Raucous rough, wild, hoarse, guttering

Expedient measure, convenient, device, maneuver

Voracious insatiable, avid, hungry, big, rapacious, greedy

Compliance fulfillment, obedience

Pedigree rare-breed, full-blooded, lineage

Diffident shy, insecure, timid

Fidelity loyalty, reliability

Plaintive mournful, sad, melancholic, nostalgic, lamenting

Augment supplement, boost, add to, bump up

Insinuate imply, suggest, make-out, ingratiate yourself

Precarious unstable, shaky, risky, uncertain

Baffle : Frustrate

Derogatory disparaging, critical, insulting, offensive

Confiscate : appropriate

Onus responsibility, burden, obligation, duty

Covet : crave

Analogous - similar, akin, related

Caprice : whim

Misdemeanor wrong, sin, crime, offense

Concur :acquiesce

Exonerate clear, forgive, absolve

Cargo :freight

Gregarious outgoing, extroverted, sociable, expressive,

Dispel : Scatter

Admonish : usurp

Divulge : reveal, disclose

Meager :scanty

Discretion: prudence

Alienate : estrange

Emancipate : liberate

Merry : gay

Efface : obliterate

Edited by: Riyaz Ali Khan Mayana

B. Tech .(CSE), D. E. C. E.

Email: riyaz666@gmail.com

Brim : Boarder

Hover : linger

obstinate : stubborn

Heap : to pile

Pretention:pretentioius

Instigate : incite

Tranquil:serene

latitude : scope

solicit : urge

latent : potential

subside : wane

lethargy : stupor

furtive :stealthy

momentary : transient i

misery : disstress

ncentive : motivation, encouragement, spur

volume :quantity

inert: passive

veer : diverge

belief : conviction

stifle : suffocate, suppress, choke

lament : wail

adhesive : bonding agent, glue, gum

to merit :to deserve


Hamper : obstruct

some antonyms

morose * lighthearted

pristine * sullied

neologism * archaism

turbid * limpid

rarefy * condense

precipitate * dilatory /contradictory

caustic * innocuous

revere * threaten

solvent * precipitant

improptu * carefully reharsed

estimable * infamous

avid * indifferent

balloon * decrease slowly

1. Irksome * pleasant
2. Jaunty * sorrowful, sad

1.Compose

x disturb

3. Nebulous * precise

2.Pristine

x sullied

4. Misapprehension * comprehension * understanding

3.Turbid x limpid

5. Obese * thin

4.Monetary

x non-economical

6. Whimsical * ordinary

5.Revere

x threaten

Compose x disturb

6.Hamper

x facilitate

Pristine x sullied

7.Transient

x permanence

Turbid x limpid

8.Fascinate

x mundane

Precipitate x dilatory/contradictory Freshersworld.com

9.Fickle x loyal

Revere x threaten

10.Contraband x legal goods

Hamper x facilitate

11.Repellent

Slur x

12.Slur x grace

Protean x

13.Protean

Fascinate x mundane

14.Hidebound x broadminded

Fickle x loyal

15.Precipitate

1)

Mollify

Enrage, Provoke

2)

Inundate

3)

Equanimity

4)

Gauche Tactful

x attractive
x constant
x dilatory/contradictory

5)

Exhume Inter

Drain

6)

Baleful

Beneficent

Agitation

7)

Anathematize

Bless

8)

Enigmatic

Unambiguous

Consensus * disagreement

9)

Pariah

Idol

Retrograde * forward looking

10) Turbid

Galleon

Paradox * consistency

Chide * praise

Stilted * natural

Depravity * goodness,righteousness

Levity * gravity

Limpid

Fritty

Word
Abysmal

Meaning
Adj. Bottomless Use : His arrogance is exceeded only

Edited by: Riyaz Ali Khan Mayana

B. Tech .(CSE), D. E. C. E.

Synonyms
Terrible, awful, dreadful,

Antonyms
Superb

Email: riyaz666@gmail.com

acronym

by his abysmal ignorance


n. A word formed by the initial letters of a multi-word

appalling, very bad,


Contraction, ellipsis,

admonish

name
v. warn strongly, reprove Use: He admonished his

Reprove, reprimand, chide,

Approve

Circumspect

listeners to change their wicked ways


v. Investigation before acting,

rebuke, usurp
Cutious, prudent, careful,

reckless

Use: She tried always to be circumspect

guarded, wary, judicious,

Conciliation

n. the act of placating

vigilant,
Appeasement, pacification,

Connotation

n. Suggested or implied meaning of an expression.

propitiation
Nuance, suggestion,

Use: Foreigners frequently are unaware of the

implication, undertone,

connotations of the words they use.


Adj. secret or hidden, not openly practiced or vowed

overtone, subtext,
Clandestine, concealed,

n. a covering that serves to conceal or shelter

stealthy, underground, copse,

something Use: Investigations of CIA reveal that

wood, thicket

such covert operations can get out of control


Adj. avaricious, eagerly desirous of Use: The child

envious, jealous, desirous,

Generous,

was covetous by nature and wanted to take the toys

greedy

temperate

belonging to his classmates


v. express disapproval of, protest against, belittle

Denounce, deplore, condemn,

Approve

Use: A firm believer of old-fashioned courtesy, Miss

censure, denigrate, decry,

Post, deprecated the modern tendency to address

belittle.

new acquaintances by their first names.


n. care and perseverance in carrying out tasks. Use :

Hard-working, assiduousness,

Laziness,

Her employers were greatly impressed by her

meticulousness,

carelessness,

diligence and offered her a partnership in the firm

conscientiousness,

negligence

n. ability to adjust action to circumstances Use: Use

painstakingness
Prudence, caution, acumen,

Indiscretion

your discretion in this matter and do not discuss the

predilection, sagacity,

dispel

matter with anyone.


v. scatter, drive away, cause to vanish Use: The

wariness, volition
Disperse

Dissemination

bright sunlight eventually dispelled the morning mist


v. Opening to public discussion or debate, the act of

Broadcasting, diffusion,

spreading something, Use:By their use of the

propagation,

covert

Covetous

Deprecate

Diligence

Discretion

Incitement

Open

Attract

internet, propagandists, have been able to


disseminate their pet doctrines to new audiences
around the globe
Adj. Opinionated Use: We tried to discourage him

Arbitrary, doctrinal, unbending,

from being so dogmatic, but nothing could convince

inflexible, authoritarian,

echelon
fallible

him that his opinions might be wrong


n. A body of troops arranged in a line
Adj. Liable to err. Use: I Know I am fallible, but I feel

Level, stratum, rank, height


Imperfect, unsound, mortal,

Harbinger

confident that I am right this time


n. an indication of the approach of something or

weak, frail,
Forerunner, herald, potent,

someone, forerunner v. fore shadow or presage Use:

omen,

illustrious

The crocus is an early harbinger of spring


Adj. Widely known and esteemed

Memorable, well-known,

Inglorious,

impetus

n. A force that moves something along, incentive,

famous
Momentum, thrust, motivation

shameful
inertia

Alternating, sporadic

Constant

Dogmatic

Flexible

Perfect

stimulus Use: A new federal highway program would


create jobs and will give added impetus to our
Intermittent

economic recovery
Adj. Periodic Use: The outdoor wedding reception had
to be shifted indoor to avoid the intermittent showers
that fell on and of all afternoon.

Edited by: Riyaz Ali Khan Mayana

B. Tech .(CSE), D. E. C. E.

Email: riyaz666@gmail.com

Latent

Adj. Potentially existing, but not presently evident or

Dormant, embryonic,

Overt

realized Use: Polaroid pictures are popular at the

suppressed, undeveloped,

parties because you can see the latent photographic


Latitude

Manifestation

mitigate

motley

pensive

prolific

image gradually appear before your eyes


n. freedom from narrow limitations Use: I think you

Leeway, freedom, autonomy,

have permitted your son too much latitude in this

liberty, room, rope

manner
n. outward demonstration, indication Use: Mozarts

Sign, demonstration,

early attraction to the harpsichord was the first

expression, symptom,

manifestation of his pronounced musical bent


v. lessen or try to lessen the seriousness or extent of

Appease, alleviate, allay,

Aggrevate,

Use: Nothing he could do to mitigate her anger, she

assuage, mollify, extenuate,

exacerbate

refused to forgive him


Adj. Multicolored or mixed Use: he wore a motley

palliate
Assorted, diversed, mixed,

Uniform,

tunic, red and green and blue and gold all patched

dissimilar,

homogenous,

together haphazardly
Adj. Dreamily thoughtful, thoughtful with a hint of

Brooding, pondering,

sadness Use: The pensive lover gazed at the portrait

meditative, contemplative

of his beloved and sighed deeply.


Adj. Abundantly fruitful Use: She was a prolific

Productive, abundant, fecund

monochrome

writer, who produced as meany as three books a


relinquish

solicit

tractable

Unproductive,
scarce

years
v. give up something with reluctance, yield Use: once

Surrender, renounce, abandon,

you get used to fringes like expense account meal

repudiate, cede,

and company car, it is very difficult to relinquish them


v. request earnestly, seek Use: The Mayor telephoned

Importune, implore, crave,

all the member of the city council to solicit their votes

beseech,

Adj. Docile, easily managed, susceptible to

Obedient, dutiful, well-

Disobedient,

suggestion Use: Although Susan seemed to be a

mannered

intractable

Vanquish, retain,

Grant

tractable young woman, she had a stubborn streak of


veer

independence
v. change in direction Use: After what seemed to be

Turn, swerve

en eternity, the wind veered to the east and the


storm abated
Vehement

Adj. Forceful, intensely emotional; with marked vigor

Fervent, passionate,

Apathetic

Use: He became so Vehement in describing what had


happened with him that he started jumping up and
Caprice

stifle

concur
lethargy

down
n. whim, a sudden desire. Use: She was an

Whim, impulse, quirk, fad,

unpredictable creature, acting on caprice, never

fancy, notion

taking thought of consequences


v. suppress, extinguish, inhibit Use: Halfway through

Smother, asphyxiate, choke,

the boring picture, Laura gave up trying to stifle he

suffocate, strangle, curb,

yawns.

restrain, repress,

v. agree Use: Did you concur with the decision of the

Agre, harmonize, coincide,

court or did you find it unfair?

assent, acquiesce

v. lack of vitality or energy Use: A complete days

Stupor, indolence, weariness

Get-up-and-go

Surreptitious, clandestine

open

Let out

Conflict, resist

work left him in a state of lethargy


Furtive

Adj. stealthy, sneaky, secret and sly or sordid Use:


noticing the furtive glance the customer gave the
diamond bracelet on the counter, the jeweler
wondered whether he had a potential shoplifter in his
hands

Edited by: Riyaz Ali Khan Mayana

B. Tech .(CSE), D. E. C. E.

Email: riyaz666@gmail.com

efface

10

v. wipe out or make dim Use: The coin had been

Obliterate, eradicate,

Engrave

Adj. Intended to attract notice and impress others,

Ostentatious, pompous,

Down-to-earth

making unjustified claims, overambitious Use: The

conceited

handled so many times that its date had been effaced


Pretentious

other prize winner isnt wearing her medal.; isnt it a


bit pretentious of you to wear yours?
compunction

n. a feeling of deep regret (usually of some misdeed)

Regret, scruple, qualm, guilt,

Use: The judge was especially sever in his sentencing

reluctance, hesitation,

as the criminal had shown no compunction for his


heinous crime

QUANTITATIVE SECTION
Aptitude
1. A family, planning a weekend trip, decides to spend not more than a total of 8 hours driving. By leaving early in the
morning, they can average 40 miles per hour on the way to their destination. Due to the heavy Sunday traffic, they can
average only 30 miles per hour on the return trip. What is the farthest distance from home they can plan to go?
(a) 120 miles or less

(b) Between 120and 140 miles

(d) Between 140 and 160 miles

(c) 140 miles

(e) 160 miles or more

2. A car is filled with four and half gallons of fuel for a round trip. If the amount of fuel taken while going is 1/4 more than
the amount taken for coming, what is the amount of fuel consumed while coming back?
a) Less than 2 gallons

(b) 2 gallons

(c) 2 1/2 gallons

(d) 3 gallons

(e) More than 3 gallons

3. A 3-gallon mixture contains one part S and two parts R. In order to change it to a mixture containing 25% S, how much R
should be added?
(a) 1/2 gallon

(b) 2/3 gallon

(c) 3/4 gallon

(d) 1 gallon

(e) 1 1/2 gallon

4. A tree grows only 3/5 as fast as the one beside it. In four years the combined growth of the two trees is eight feet.
How much does the shorter tree grow in two years?
a) Less than 2 feet

(b) 2 feet

(c) 2 1/2 feet

(d) 3 feet

(e) more than 3 feet.

5. Wind flows at 160 miles in 330 minutes, for traveling 80 miles how much time does it require?
(a) 1 hour 30 minutes

(b) 1 hour 45 minutes

c) 2 hours

(d) 2 hours 45 minutes

(e) 3 hours

6. A stationary engine has enough fuel to run 12 hours when its tank is 4/5 full. How long will it run when the tank is 1/3 full?
(a) Less than 2 hours

(b) 2 hours

(c) 3 hours

(d) 4 hours

(e) 5 hours

7. If A is traveling at 72 km per hour on a highway. B is traveling at a speed of 25 meters per second on a highway. What is
the difference in their speeds in meters per second?
(a) 1/2 m/sec

(b) 1m/sec

(c) 1 1/2 m/sec

(d) 2 m/sec

(e) 3 m/sec

8. A salesperson by mistake multiplied a number and got the answer as 3, instead of dividing the number by 3. What is the
answer he should have actually got?
(a) 0

(b) 1/3

(c) 1

(d) 2

(e) 3

9. If the length of a rectangle is increased by 30% and the width is decreased by 20%, then the area is increased by...
(a) 10%

(b) 5%

(c) 4%

(d) 20%

(e) 25%

10. In the class of 40 students, 30 speak Hindi and 20 speak English. What is the lowest possible number of students who
Edited by: Riyaz Ali Khan Mayana B. Tech .(CSE), D. E. C. E.
Email: riyaz666@gmail.com

11

speak both the languages?


(a) 5

(b) 20

(c) 15

(d) 10

(e) 30

11. The most economical prices among the following prices is:
(a) 10 kilo for Rs.160

(b) 2 kilo for Rs.30

(d) 20 kilo for Rs.340

(e) 8 kilo for Rs.130

(c) 4 kilo for Rs.70

12. A truck contains 150 small packages, some weighing 1 kg each and some weighing 2 kg each. how many packages
weighing 2 kg each are in the truck if the total weight of all the packages is 264 kg?
(a) 36

(b) 52

(c) 88

(d) 124

(e) 114

13. A man was arrested for exceeding the speed limit by 10 miles an hour. A second man was charged with exceeding the
same limit by twice as much. The second man was driving 35 miles per hour. What was the speed limit?
(a) 10 miles per hour

(b) 15 miles per hour

(d) 25 miles per hour

(e) 30 miles per hour

(c) 20 miles per hour

14. One year ago Pandit was three times his sister's age. Next year he will be only twice her age. How old will Pandit be after
five years?
(a) 8

(b) 12

(c) 11

(d) 13

(e) 15

15. If two pencils cost 8 cents, then how much do 5 pencils cost?
(a) 18 cents

(b) 20 cents

(c) 22 cents

(d) 23 cents

(e) 24 cents

1. WHICH IS THE NEXT NO:


5,6,7,8,10,11,14,?
2. BFGE CODED AS CEHD

ANS: 18
THEN CODE

PVHDJ. ANS: QUICK

3. FIND THE NO. OF Y FOLLOWED BY W BUT THAT IS NOT


FOLLOWED BY Z.
Y W R U D D Y W Z ...............
4. WHAT IS THE LARGEST PRIME NO THAT IS STORED IN 8
BIT PATTERN.

ANS: 253 (NOT SURE)

5.WHICH WILL GIVE GOOD STANDARD DEVIATION


1. (7,0,-7,0,7)

2. (7,-7,7,-7,7)

3. (1,0,-1,0,1)

6. WHICH IS NOT A SIDE OF A RECTANGULAR


1. (2,3,4)

2.(3,4,7)

3. (3,5,9)

7.WHICH SHAPE WILL BE OBTAINED BY USING THESE VALUES


OF X ,Y
X

0.00001

10

1.02

100

1.72

1000

3.00

9999

4.72

8. WHICH EQUATION THAT BEST SUITS THIS CURVE


A LINE CUTS X AT -1 WHEN Y=0 AND X=0 WHEN Y=3 AND
GOES UPWARD
Edited by: Riyaz Ali Khan Mayana

B. Tech .(CSE), D. E. C. E.

Email: riyaz666@gmail.com

12

Y
X
9. A MAN , WOMAN AND A BOY JOINDLY DID A JOB IN 6 DAYS. A MAN ALONE FINISHES IN 10 dAYS, A WOMEN ALONE
FINISH IN 24 DAYS. THEN HOW MANY DAYS THE BOY CAN TAKE TO FINSH?
10. FOR TEMPERATURE A FUNCTION IS GIVEN ACCORDING TO TIME : ((t**2)/6) + 4t +12

WHAT IS THE TEMPERATURE

RISE OR FALL BETWEEN 4.AM TO 9 AM


11. AN AEROPLANE STARTS FROM A (SOME LATITUDE IS GIVEN ACCORDING TO PLACE)AT 2 AM lOCAL TIME TO B(SOME
LATITUDE). TRAVELLLING TIME IS 10 HOURS. WHAT IS THE LOCAL TIME OF B WHEN IT REACHES B
12. A FILE IS TRANSFERRED FROM A PLACE TO ADESTINATION CAPABLE OF 10 KB . THEY GIVEN SOME RATE OF
TRANSFER. U HAVE FIND A EQUATION THAT BEST SUIT THIS.
13. IN A PLANAR CUBE , THE NO. OF VERTICES, NO OF EDGES AND NO OF FACES ARE
1. 6,6,6 2. 4,8,12

3....

4.........

14. VENN DIAGROM below


1.

HOW MANY PERSON KNOW ENGLISH MORE THAN FRENCH

2.

HOW MUCH % OF PEOPLE KNOWS ALL THE 3 LANGUAGES

3.

HOW MUCH % OF PEOPLE THOSE WHO KNOWS FRENCH AND

GERMAN AND NOT ENGLISH


FRENCH

15. CORRECT CHART WITH CORRECT VALUES


X- YEAR
Y- NO OF PERSONS
1.

AVERAGE NO. OF PERSONS FROM 1995 - 1999

2.

WHICH YEAR HAS LARGE DIFFERENCES IN NO OF PERSONS

3.

IF 10% OF PEOPLE LEAVES THE OFFICE IN 1998 THEN,

HOW MANY FRESH CANDIDATES CAN BE ADDED IN THE NEXT


YEAR
16. WHAT IS THE VALUE OF M(373, 7) + R(6.8) -T(3.4) + R( 3.4)

M- MODULAS

R- ROUNDOFF

17. WHAT IS THE VALUE OF

% - DOUBLING

#- RECIPROCAL

% # % (5) + # % # (2)

WHERE

T- TRUNCATE

18. MATCH THE FOLLOWING


A

1. SENTENCE, PARAGRAPH

1. TYPE OF

2. BASMATI, WHEAT

2. A PART OF

3. BROTHER, SISTER

3. NOT A TYPE OF

4. BREIGAL, DOG

4. SIBLING

ANS: 1-> 2

2->1

3->3

4->4

19. G (0) =1 G (1)= -1 IF G (N)=2* (G (N-1))- 3(G(N-2)) THEN WHAT IS THE VALUE OF G (4)?
Edited by: Riyaz Ali Khan Mayana

B. Tech .(CSE), D. E. C. E.

Email: riyaz666@gmail.com

20.

13

(AUB)PC
21.

TIME

22.

DEGREE

7 6' 43.15"

5 31' 4.3"

THEN WHAT WILL BE THE DEGREE WHEN TIME IS 3 O CLOCK

THREE COMPANIES WORKING INDEPENDENTLY AND GET SAVINGS 10%, 20%, 25%. IF THE COMPANIES WORK

JUST OPPOSITE THEN WHAT WILL BE THE NET SAVING?


23. WHICH ONE WILL BE THE EXACT POWER OF 3

(i) 2768

(ii)2678

(III) 2187

24. SOME RELATION THAT IS DEDUCE TO


A (POW 2) DIRECT PROPORTIONAL TO X (POW 3)
B (POW 2) DIRECT PROPORTIONAL TO Y (POW 3)
SOME FOUR ANSWERS WERE GIVEN
ANS: ALL OF THE ABOVE
25. 900 M WIDE 3000 M WIDTH
SOMETHING I CAN'T REMEMBER SOME VALUES ARE GIVEN
BY AIR PER M Rs. 4

BY GROUND PER M Rs. 5

THEN WHERE U WILL CUT ADD MAN OUT


26. 1.JAVA

2.SMALLTALK

3.LISP

27. 1.SAP

2.ARP

3.WAP

4.EIFFEL
4.TCP IP

28. WHICH IS THE PERFECT ONE AMONG THE 4


1. 2x +3y=4
1.

2. x + y= -1

3. Y=2x+ 3

5,6,7,8,10,11,14?

2.
A

(AuB)nC = ?
3.

[(A union B) intersection C =?]

Find the Odd word.

Java, Lisp, Smalltalk, Eiffel


(One more question of same type - Odd man out)
4.
Year

95

Members 100

96

97
1

70

98

99

10

50

i ) Which year has maximum members growth?

Two more questions on this. Please go

Through Barons GRE data interpretation problems

Edited by: Riyaz Ali Khan Mayana

B. Tech .(CSE), D. E. C. E.

Email: riyaz666@gmail.com

5.

In Madras, temperature at noon varies according to

14
-t^2/2 + 8t + 3 (READ as: -t square /2 +...), where t is elapsed

time. Find how much temp. More or less in 4pm to 9pm. (May be we can solve it by Definite Integration. Check any way)
6.

A man, a woman, and a child can do a piece of work in 6 days. Man only can do it in 24 days. Woman can do it in 16

days and in how many days child can do the same work? (Numbers are not correct. Problem model is important)
7.

What is the highest prime number that can be stored in a 8-bit microprocessor?

8.

In which of the system 384 is equal to 1234? (Numbers are not correct. Important is only problem model)

9.

If D_MUQZM is coded as CENTRAL then RBDJK can be coded as ---------

10.

The size of a program is N. And the memory occupied by the program is given by M = square root of 100N. If the

size of the program is increased by 1% then how much memory now occupied?
11. French, English, German

1.How

many more or less speak English than French?


2.What % people speak all the three languages?
3.What % people speak German but not English?
12.

The size of the bucket is N KB. The bucket fills at the rate of 0.1 KB per millisecond. A programmer sends a program

to receiver. There it waits for 10


Milliseconds. And response will be back to programmer in 20 milliseconds.
Based on above information one question is there.
13.

A power unit is there by the bank of the river 750 mts. A cable is made from power unit to power a plant opposite to

that of the river at 1500mts. The cost of the cable below water is Rs. 15/- per meter ansd cost of cable on the bank is Rs.
12/- per meter.
Based on above information one question is there.
14.
i.

Match the following.


brother - sister

a. Part of

ii. alsecian - dog

b. Sibling

iii. sentence - paragraph

c. Type of

iv.

d.

15.

If the vertex (5,7) is placed in the memory. First vertex (1,1) 's address is 1245 and then address of (5,7) is -----

16.

A Planar solid cube contains how many vertices, how many corner points and how many faces?

17.

Which of the equation satisfies the graph? 4 equations given.

18.

Square of the <<term>> is directly proportional to the cube of the <<another term>> . If the <<term>> is 'a' and

<<another term>> is 'b' then which one is correct?


4 options given like a square / b cube =const.
QUANTITATIVE SECTION
1. If two pencils cost 8 cents, then how much do 5 pencils cost?
Ans. 20 cents

Edited by: Riyaz Ali Khan Mayana

B. Tech .(CSE), D. E. C. E.

Email: riyaz666@gmail.com

15

2. Some work is done by two people in 24 minutes. One of them can do this work alone in 40 minutes. How much time does
the second person take to do the same work ?
Ans. 60 minutes
3. A car is filled with four and half gallons of fuel for a round trip.If the amount of fuel taken while going is 1/4 more than the
amount taken for coming, what is the amount of fuel consumed while coming back?
Ans.2 gallons
4. The lowest temperature in the night in a city A is 1/3 more than 1/2 the highest during the day. Sum of the lowest
temperature and the highest temperature is 100 degrees. Then what is the low temp?
Ans.40 degrees
5. Javagal, who decided to go to weekened trip should not exceed 8 hours driving in a day. The average speed of forward
journey is 40 miles/hr.Due to traffic on sundays, the return journey's average speed is 30 m/h. How far he can select a picnic
spot?
a) 120 miles
b) between 120 and 140 miles
c) 160 miles
Ans. 120 miles
6. A salesperson by mistake multiplied a number and got the answer as 3, instead of dividing the number by 3.What is the
answer he should have actually got?
Ans. 3
7. A building with height D shadow upto G. What is the height of a neighbouring building with a shadow of C feet.
Ans. (C*D)/G
8. A person was fined for exceeding the speed limit by 10 mph. Another person was also fined for exceeding the same speed
limit by twice the same. If the second person was travelling at a speed of 35 mph, find the speed limit.
Ans. 15 mph
9. A bus started from bustand at 8.00am, and after staying for 30 minutes at a destination, it returned back to the busstand.
The destination is 27 miles from the busstand. The speed of the bus is 18mph. During the return journey bus travels with
50% faster speed.At what time does it return to the busstand?
Ans. 11.00am
10. In a mixture, R is 2 parts and S is 1 part. In order to make S to 25% of the mixture, how much of R is to be added?
Ans.One part of R
11. Wind flows 160 miles in 330 min, for travelling 80 miles how much time does it require?
Ans. 2 hrs 45 mins
12. With a 4/5 full tank a vehicle can travel 12 miles, how far can it travel with a 1/3 full tank
Ans. 5 miles
13. There are two trees in a lawn. One grows at a rate 3/5 of the other in 4 years. If the total growth of trees is 8 ft. What is
the height of the smaller tree after 2 years
Ans. 1 1/2 feet

Edited by: Riyaz Ali Khan Mayana

B. Tech .(CSE), D. E. C. E.

Email: riyaz666@gmail.com

16

14. Refer to the figure below.A ship started from P and moves at a speed of I miles per hour and another ship starts from L
and moving with H miles per hour simultaneously.Where do the two ships meet?
||---g---||---h---||---i---||---j---||---k---||---l---||
PG H I J K L are the various stops in between denoted by || . The values g, h, i, j, k, l denote the distance between the ports.
Ans. Between I and J, closer to J
15. If A is travelling at 72 km per hour on a highway. B is travelling at a speed of 25 meters per second on a highway. What is
the difference in their speeds in m/sec.
Ans. 1 m/sec

1. There are 150 weights .Some are 1 kg weights and some are 2 kg weights. The sum of the weights is 260.What is the
number of 1kg weights?
Ans. 40
2. A is driving on a highway when the police fines him for overspeeding and exceeding the limit by 10 km/hr.At the same time
B is fined for overspeeding by twice the amount by which A exceeded the limit.If he was driving at 35 km/hr what is the
speed limit for the road?
Ans. 15 kmph
3. A moves 3 kms east from his starting point . He then travels 5 kms north. From that point he moves 8 kms to the
east.How far is A from his starting point?
Ans. 13 kms
4. A car travels 12 kms with a 4/5th filled tank.How far will the car travel with 1/3 filled tank?
Ans. 5 kms
5. The sum of the digits of a two digit number is 8. When 18 is added to the number, the digits are reversed. Find the
number?
Ans. 35
6. The cost of one pencil, two pens and four erasers is Rs.22 while the cost of five pencils, four pens and two erasers is
Rs.32.How much will three pencils, three pens and three erasers cost?
Ans. 27
7. Fathers age is 5 times his son's age. 4 years back the father was 9 times older than son.Find the fathers' present age.
Ans. 40 years
8. What number should be added to or subtracted from each term of the ratio 17 : 24 so that it becomes equal to 1 : 2.
Ans. 10 should be subtracted
9. What is the 12th term of the series 2, 5, 8, ....
Ans. 35
10. If 20 men take 15 days to to complete a job, in how many days can 25 men finish that work?
Ans. 12 days
11. In a fraction, if 1 is added to both the numerator at the denominator, the fraction becomes 1/2. If numerator is
subtracted from the denominator, the fraction becomes 3/4. Find the fraction.
Edited by: Riyaz Ali Khan Mayana

B. Tech .(CSE), D. E. C. E.

Email: riyaz666@gmail.com

17

Ans. 3/7
12. If Rs.1260 is divided between between A, B and C in the ratio 2:3:4, what is C's share?
Ans. Rs. 560
13. A shopkeeper bought a watch for Rs.400 and sold it for Rs.500.What is his profit percentage?
Ans. 25%
14. What percent of 60 is 12?
Ans. 20%
15. Hansie made the following amounts in seven games of cricket in India: Rs.10, Rs.15, Rs.21, Rs.12, Rs.18, Rs.19 and
Rs.17(all figures in crores of course).Find his average earnings.
Ans. Rs.16 crore

1) ONE RECTANGULAR PLATE WITH LENGTH 8INCHES, BREADTH 11 INCHES AND


2 INCHES THICKNESS IS THERE.WHAT IS THE LENGTH OF THE CIRCULAR ROD
WITH DIAMETER 8 INCHES AND EQUAL TO VOLUME OF RECTANGULAR PLATE?
ANS: 3.5 INCHES
2) WHAT IS THE NUMBER OF ZEROS AT THE END OF THE PRODUCT OF THE NUMBERS
FROM 1 TO 100

3) In some game 139 members have participated every time one fellow will get bye what is the number of matches to choose
the champion to be held?
ans: 138
4) one fast typist type some matter in 2hr and another slow typist type the same matter in 3hr. if both do together in how
much time they will finish.
ans: 1hr 12min
5) in 8*8 chess board what is the total number of squares refer Model
ans:204
6) falling height is proportional to square of the time. one object falls 64cm in 2sec than in 6sec from how much height the
object will fall.
7) Gavaskar average in first 50 innings was 50. After the 51st innings his average was 51 how many runs he made in the 51st
innings
8)2 oranges, 3 bananas and 4 apples cost Rs.15. 3 oranges 2 bananas 1 apple costs Rs.10. what is the cost of 3 oranges, 3
bananas and 3 apples?
Ans: Rs.15
9) In 80 coins one coin is counterfeit what is minimum number of weighing to find out counterfeit coin
10)In a company 30% are supervisors and 40% employees are male. if 60% of supervisors are male. What is the probability
that a randomly chosen employee is a male or female?
11) statement: all green are blue are blue, all blue are white
conclusion:
Edited by: Riyaz Ali Khan Mayana

B. Tech .(CSE), D. E. C. E.

Email: riyaz666@gmail.com

18

I) some blue are green II) some white are green


III)some green are not white IV) all white are blue
a) he has given four choices like gre type
12)all teachers are students. some students are girls.this type of questions are there. we cant able to reproduce them.

QUANTITATIVE AND LOGICAL REASONING

1. In a two-dimensional array, X (9, 7), with each element occupying 4 bytes of memory, with the address of the first
element X (1, 1) is 3000; find the address of X (8, 5).
Ans: 3212

2. In the word ORGANISATIONAL, if the first and second, third and forth, forth and fifth, fifth and sixth words are
interchanged up to the last letter, what would be the tenth letter from right?
Ans:I

3. What is the largest prime number that can be stored in an 8-bit memory? Ans : 251

4. Select the odd one out..a. Java b. Lisp c. Smalltalk d. Eiffel.


5. Select the odd one out a. SMTP b. WAP c. SAP d. ARP
6. Select the odd one out a. Oracle b. Linux c. Ingress d. DB2
7. Select the odd one out a. WAP b. HTTP c. BAAN d. ARP
8. Select the odd one out a. LINUX b. UNIX c. SOLARIS d. SQL SEVER
9. Select the odd one out a. SQL b. DB2 c. SYBASE d. HTTP

10. The size of a program is N. And the memory occupied by the program is given by M = square root of 100N. If the size of
the program is increased by 1% then how much memory now occupied?

Ans:

0.5%(SQRT 101N)

11. A man, a woman, and a child can do a piece of work in 6 days. Man only can do it in 24 days. Woman can do it in 16 days
and in how many days child can do the same work?
Ans: 16
12. In which of the system, decimal number 184 is equal to 1234?

13. Find the value of the 678 to the base-7.

Ans: 1656

14. Number of faces, vertices and edges of a cube

Ans: 6 8 12

15. Complete the series 2, 7, 24, 77,__

Ans: 238

Ans: 5

16. Find the value of @@+25-++@16, where @ denotes "square" and + denotes "square root".
Ans: 621

17. Find the result of the following expression if, M denotes modulus operation, R denotes round-off, T denotes truncation:
M(373,5)+R(3.4)+T(7.7)+R(5.8)

Ans:19

18. If TAFJHH is coded as RBEKGI then RBDJK can be coded as? Ans: qcckj

19. G(0)= -1, G(1)=1, G(N)=G(N-1) - G(N-2), G(5)= ?


Edited by: Riyaz Ali Khan Mayana

B. Tech .(CSE), D. E. C. E.

Ans: - 2
Email: riyaz666@gmail.com

19

20. What is the max possible 3 digit prime number?

Ans: 997

21. A power unit is there by the bank of the river of 750 meters width. A cable is made from power unit to power plant
opposite to that of the river and 1500mts away from the power unit.The cost of the cable below water is Rs.15/- per meter
and cost of cable on the bank is Rs.12/-per meter. Find the total of laying the cable.

Ans : 1000 (24725-cost)

22. The size of a program is N. And the memory occupied by the program is given by M = square root of 100N. If the size of
the program is increased by 1% then how much memory now occupied?
Ans:0.5%(SQRT 101N)

23. In Madras, temperature at noon varies according to -t^2/2 + 8t + 3, where t is elapsed time. Find how much
temperature more or less in 4pm to 9pm. Ans:At 9pm 7.5 more

24. The size of the bucket is N kb. The bucket fills at the rate of 0.1 kb per millisecond. A programmer sends a program to
receiver. There it waits for 10 milliseconds. And response will be back to programmer in 20 milliseconds. How much time the
program takes to get a response back to the programmer, after it is sent?

Ans: 30

25. A man, a woman, and a child can do a piece of work in 6 days. Man only can do it in 24 days. Woman can do it in 16 days
and in how many days child can do the same work?
Ans: 16

26. If the vertex (5,7) is placed in the memory. First vertex (1,1) `s address is 1245 and then address of (5,7) is ---------Ans: 1279

27. Which of the following are orthogonal pairs?


a. 3i+2j b. i+j c. 2i-3j d. -7i+j

Ans: a, c

28. If VXUPLVH is written as SURMISE, what is SHDVD?

Ans: PEASE

29. If A, B and C are the mechanisms used separately to reduce the wastage of fuel by 30%, 20% and 10%.What will be the
fuel economy if they were used combined.
Ans: 20%

30. What is the power of 2?

a. 2068

b.2048 c.2668

31. Complete the series. 3, 8, --, 24, --, 48, 63.

Ans: 15.35

32. Complete the series. 4, -5, 11, -14, 22, --

Ans: -27

33. A, B and C are 8 bit no's. They are as follows:


A -> 1 1 0 1 1 0 1 1
B -> 0 1 1 1 1 0 1 0
C -> 0 1 1 0 1 1 0 1
Find ((A-B) u C)=?

Hint: 109. A-B is {A} - {A n B}

34. A Flight takes off at 2 A.M from northeast direction and travels for 11 hours to reach the destination, which is in
northwest direction. Given the latitude and longitude of source and destination. Find the local time of destination when the
flight reaches there? Ans: 7 am
Edited by: Riyaz Ali Khan Mayana

B. Tech .(CSE), D. E. C. E.

Email: riyaz666@gmail.com

20

35. A can copy 50 papers in 10 hours while both A & B can copy 70 papers in 10 hours. Then for how many hours required
for B to copy 26 papers?

Ans: 13

36. A is twice efficient than B. A and B can both work together to complete a work in 7 days. Then find in how many days, A
alone can complete the work? Ans: 10.5

37. A finish the work in 10 days. B is 60% efficient than A. So how many days do B takes to finish the work?
Ans :100/6

38. A finishes the work in 10 days & B in 8 days individually. If A works for only 6 days then how many days should B work to
complete A's work?

Ans: 3.2 days

39. Given the length of the 3 sides of a triangle. Find the one that is impossible? (HINT: sum of smaller 2 sides is greater
than the other one, which is larger)

40. Find the singularity matrix from a given set of matrices? (Hint det(A)==0))

41. A 2D array is declared as A[9,7] and each element requires 2 byte. If A [1,1] is stored in 3000. Find the memory of
A[8,5] ?

Ans: 3106

42. Sum of slopes of 2 perpendicular st.lines is given. Find the pair of lines from the given set of options, which satisfy the
above condition?

43. (a) 2+3i (b)1+i (c) 3-2i (d) 1-7i .Find which of the above is orthogonal. Ans : a,c

44. (Momentum*Velocity)/(Acceleration * distance). Find units.

Ans: mass

45. The number 362 in decimal system is given by (1362) x in the X System of numbers find the value of X
a} 5

b) 6

c) 7

d) 8

e) 9

46. Given $ means Tripling and % means change of sign then find the value of $%$6-%$%6

47. My flight takes of at 2am from a place at 18N 10E and landed 10 Hrs later at a place with coordinates 36N70W. What is
the local time when my plane landed?
6:00 am

b) 6:40am

c) 7:40

d) 7:00

e) 8:00

(Hint: Every 1 deg longitude is equal to 4 minutes. If west to east add time else subtract time)

48. Find the highest prime number that can be stored in an 8 bit Computer?

49. Which of the following set of numbers has the highest Standard deviation?
a)1,0,1,0,1,0

b) -1, -1, -1, -1, -1, -1

c)1,1,1,1,1,1

d) 1,1,0, -1,0, -1

1. My father has no brothers. he has three sisters who has two Childs each.
1> my grandfather has two sons (f)
2> three of my aunts have two sons (can't say)
3> my father is only child to his father (f)
Edited by: Riyaz Ali Khan Mayana

B. Tech .(CSE), D. E. C. E.

Email: riyaz666@gmail.com

21

4> I have six cousins from my mother side (f)


5> I have one uncle (f)
2. Ether injected into gallbladder to dissolve gallstones. This type one-day treatment is
enough for gallstones not for calcium stones. This method is alternative to surgery for millions of people who are suffering
from this disease.
1> calcium stones can be cured in one day (f)
2> hundreds of people contains calcium stones(can't say)
3> surgery is the only treatment to calcium stones(t)
4> either will be injected into the gallbladder to cure the cholesterol
based gall stones(t).
3. Hacking is illegal entry into other computer. This is done mostly because of lack of knowledge of computer networking with
networks one machine can access to another machine. Hacking go about without knowing that each network is accredited to
use network facility.
1> Hacking people never break the code of the company which they
work for (can't say).
2> Hacking is the only vulnerability of the computers for the usage
of the data.(f)
3> Hacking is done mostly due to the lack of computer knowledge (f).
(there will be some more questions in this one )
4. Alpine tunnels are closed tunnels. In the past 30 yrs not even a single accident has been recorded for there is one accident
in the railroad system. Even in case of a fire accident it is possible to shift the passengers into adjacent wagons and even the
living fire can be detected and extinguished with in the duration of 30 min.
1> no accident can occur in the closed tunnels (True)
2> fire is allowed to live for 30 min. (False)
3> All the care that travel in the tunnels will be carried by rail
shutters.(t)
4>
5. In the past helicopters are forced to ground or crash because of the formation of the ice on the rotors and engines. A new
electronic device has been developed which can detect the water content in the atmosphere and warns the pilot if the
temperature is below freezing temp about the formation of the ice on the rotors and wings.
1> the electronic device can avoid formation of the ice on the wings (False).
2> There will be the malfunction of rotor & engine because of formation of ice (t)
3> The helicopters are to be crashed or down (t)
4> There is only one device that warn about the formation of ice (t).
6.In the survey conducted in mumbai out of 63 newly married house wives not a single house wife felt that the husbands
should take equal part in the household work as they felt they loose their power over their husbands. In spite of their careers
they opt to do the kitchen work themselves after coming back to home. The wives get half as much leisure time as the
husbands get at the weekends.
1> housewives want the husbands to take part equally in the household (f)
2> wives have half as much leisure time as the husbands have (f)
3> 39% of the men will work equally in the house in cleaning and washing
Edited by: Riyaz Ali Khan Mayana

B. Tech .(CSE), D. E. C. E.

Email: riyaz666@gmail.com

22

7. Copernicus is the intelligent. In the days of Copernicus the transport and technology development was less & it took place
weeks to communicate a message at that time. Where in we can send it through satellite with in no time ----------. Even with
these fast developments it has become difficult to understand each other.
1> people were not intelligent during Copernicus days (f).
2> Transport facilities are very much improved in now a days (can't say)
3> Even with the fast developments of the technology we can't live happily.(can't say)
4> We can understand the people very much with the development of communication (f).
Q8) senior managers warned the workers that because of the introductory of Japanese industry in the car market. There is
the threat to the workers. They also said that there will be the reduction in the purchase of the sales of car in public. The
interest rates of the car will be increased with the loss in demand.
1> Japanese workers are taking over the jobs of Indian industry (false)
2> managers said car interests will go down after seeing the raise in interest rates (true)
3> Japanese investments are ceasing to end in the car industry (false)
4> people are very much interested to buy the cars (false)
Q9) In the totalitarian days, the words have very much devalued. In the present day, they are becoming domestic that is
the words will be much more devalued. In those days, the words will be very much affected in political area. But at present,
the words came very cheap .we can say they come free at cost.
1> totalitarian society words are devalued.(false)
2> totalitarian will have to come much about words (t)
3> The art totalitarian society the words are used for the political speeches.
4>
Q10) There should be copyright for all arts. The reele has came that all the arts has come under one copy right
society,they were use the money that come from the arts for the developments . There may be a lot of money will come from
the Tagore works. We have to ask the benifiters from Tagore work to help for the development of his works.
1> Tagore works are came under this copy right rule.(f)
2> People are free to go to the

because of the copy right rule.(can't say)

3> People gives to theater and collect the money for development.(can't say)
4> We have ask the Tagore resedents to help for the developments of art.(can't say)

TCS main

Papers From Campus Recruitment at NIT BHOPAL 2003

1.If g (0)=g (1)=1


And g (n)= g (n-1) + g (n -2) find g (6);
2.A plane moves from 9N40E to 9N40W. If the plane starts at 10 am and takes 8 hours to reach the destination, find the
local arrival time.
3. If log 0.317= and log 0.318=. Then find the value of log 0.319.
4.You will be given the bit position values for A, B and C and using the relation (AB) u C you have to construct the truth
table. Then find the corresponding decimal number and choose the right option.
5.Complete the sequence 9,10,11,13,15, __, 21,28.
6.In a certain format TUBUJPO is coded as STATION. The code of which string is FILTER?
Edited by: Riyaz Ali Khan Mayana

B. Tech .(CSE), D. E. C. E.

Email: riyaz666@gmail.com

23

7. What is the code formed by reversing the First and second letters, the third and fourth letters and son on of the string
SIMULTANEOUSLY?
8.The base 5 representation of the decimal number 2048 is _____.
9.Which is the largest prime number that can be stored in a 9-bit register?
10.Find the physical quantity represented by MOMENTUM *VELOCITY] / [LENGTH * ACCELERATION]?
11.A can do a piece of work in 20 days, which B can do in 12 days. In 9 days B does of the work. How many days will A
take to finish the remaining work?
ANNA University-Campus Recruitment July2003
QUANTITATIVE AND LOGICAL REASONING.
1. In a two-dimensional array, X (9, 7), with each element occupying 4 bytes of memory, with the address of the first
element X (1, 1) is 3000, find the address of X (8, 5).
ANS: 3212
2. In the word ORGANISATIONAL, if the first and second, third and forth, forth and fifth, fifth and sixth words are
interchanged up to the last letter, what would be the tenth letter from right?
ANS: I(ROANISATIONALG)
2E. In the word ORGANISATIONAL, if the first and second, third and forth, fifth and sixth words are interchanged up to the
last letter, what would be the tenth letter from right?
ANS: I(ROAGINASITNOLA)
3. What is the largest prime number that can be stored in an 8-bit memory?ANS:127
4. Select the odd one out. a. Java b. Lisp c. Smalltalk d.Eiffel .ANS: LISP
5. Select the odd one out a. SMTP b. WAP c. SAP d. ARP ANS: SAP
6. Select the odd one out a. Oracle b. Linux c. Ingress d. DB2 ANS:LINUX
7. Select the odd one out a. WAP b. HTTP c. BAAN d. ARP ANS:BAAN
8. Select the odd one out a. LINUX b. UNIX c.SOLARIS d. SQL SERVER ANS:SQL SERVER
9. Select the odd one out a. SQL b. DB2 c.SYBASE d. HTTp ANS:HTTP
10. The size of a program is N. And the memory occupied by the program is given by M = square root of 100N. If the size of
the program is increased by 1% then how much memory now occupied?
11. A man, a woman, and a child can do a piece of work in 6 days. Man only can do it in 24 days. Woman can do it in 16 days
and in how many days child can do the same work? ANS:16
12. In which of the system, decimal number 194 is equal to 1234? ANS:5
13. Find the value of the 678 to the base 7. ANS:1656
14. Number of faces, vertices and edges of a cube ANS:6,8,12
15. Complete the series 2, 7, 24, 77,__ ANS:238
16. Find the value of @@+25-++@1..., where @ denotes "square" and + denotes "square root". ANS:121
17. Find the result of the following _expression if, M denotes modulus operation, R denotes round-off, T denotes truncation:
M(373,5)+R(3.4)+T(7.7)+R(5.8) ANS:19
18. If TAFJHH is coded as RBEKGI then RBDJK can be coded as --------- ANS:PCCKJ
19. G(0)=-1, G(1)=1, G(N)=G(N-1) - G(N-2), G(5)= ? ANS:-2
20. What is the max possible 3 digit prime number?
ANS:
21. A power unit is there by the bank of the river of 750 meters width. A cable is made from power unit to power a plant
opposite to that of the river and 1500mts away from the power unit. The cost of the cable below water is Rs. 15/- per meter
and cost of cable on the bank is Rs.12/- per meter. Find the total of laying the cable.
ANS:20250
22. The size of a program is N. And the memory occupied by the program is given by M = square root of 100N. If the size of
the program is increased by 1% then how much memory now occupied?
23. In Madras, temperature at noon varies according to -t^2/2 + 8t + 3, where t is elapsed time. Find how much
temperature more or less in 4pm to 9pm. ANS: 385.8(DB)
24. The size of the bucket is N kb. The bucket fills at the rate of 0.1 kb per millisecond. A programmer sends a program to
Edited by: Riyaz Ali Khan Mayana

B. Tech .(CSE), D. E. C. E.

Email: riyaz666@gmail.com

24

receiver. There it waits for 10 milliseconds. And response will be back to programmer in 20 milliseconds. How much time the
program takes to get a response back to the programmer, after it
is sent? ANS: 30MILISECOND
25. A man, a woman, and a child can do a piece of work in 6 days. Man only can do it in 24 days. Woman can do it in 16
days and in how many days child can do the same work?
26. If the vertex (5,7) is placed in the memory. First vertex (1,1) ?s address is 1245 and then address of (5,7) is ---------27. Which of the following are orthogonal pairs?
a. 3i+2j b. i+j
c. 2i-3j d. -7i+j
ANS: (A)& (C).
28. If VXUPLVH is written as SURMISE, what is SHDVD? ANS: PEASA
29. If A, B and C are the mechanisms used separately to reduce the wastage of fuel by 30%, 20% and 10%. What will be the
fuel economy if they were used combined. ANS: 20%
30. What is the power of 2? a. 2068 b.2048 c.2668 ANS: (B). 2048
31. Complete the series. 3, 8, --, 24, --, 48, 63 ANS: 15,35
32. Complete the series. 4, -5, 11, -14, 22, --- ANS: -27
33. A, B and C are 8 bit no?s. They are as follows:
A11011011
B01111010
C01101101
Find ( (A-B) u C )=?
Hint :
A-B is {A} - {A n B}
ANS: 0 1 1 1 1 1 1 1 (DB)
A Flight takes off at 2 A.M from northeast direction and travels for 11 hours to reach the destination which is in north west
direction.Given the latitude and longitude of source and destination. Find the
local time of destination when the flight reaches there?
ANS: 1:00 P.M
35. A can copy 50 papers in 10 hours while both A & B can copy 70 papers in 10 hours. Then for how many hours required
for B to copy 26 papers? ANS: 13
36. A is twice efficient than B. A and B can both work together to complete a work in 7 days. Then find in how many days A
alone can complete the work? ANS: 10.5 DAYS(11)
37. A finish the work in 10 days. B is 60% efficient than A. So hoW days does B take to finish the work?ANS : 4DAYS.
38. A finishes the work in 10 days & B in 8 days individually. If A works for only 6 days then how many days should B work to
complete A?s work? ANS : 3.2 DAYS(4)
39. Given the length of the 3 sides of a triangle. Find the one that is
impossible? (HINT : sum of smaller 2 sides is greater than the other one which is larger)
40. Find the singularity matrix from a given set of matrices?(Hint det(A)==0)
41. A 2D array is declared as A[9,7] and each element requires 2 byte.If A[ 1,1
] is stored in 3000. Find the memory of A[8,5] ? ANS: 3106.
42. Sum of slopes of 2 perpendicular st. lines is given. Find the pair of lines from the given set of options which satisfy the
above condition?
43. (a) 2+3i (b)1+i (c) 3-2i (d) 1-7i .Find
which of the above is orthogonal.
ANS : (A) & (C).
44. (Momentum*Velocity)/(Acceleration * distance ) find units. ANS:MASS
45. The number 362 in decimal system is given by (1362)x in the X system of numbers find the value of X a}5 b) 6 c) 7 d) 8
e) 9
Edited by: Riyaz Ali Khan Mayana

B. Tech .(CSE), D. E. C. E.

Email: riyaz666@gmail.com

25

46. Given $ means Tripling and % means change of sign then find the value of $%$6-%$%6
ANS : -72
47. My flight takes of at 2am from a place at 18N 10E and landed 10 Hrs later at a place with coordinates 36N70W. What is
the local time when my plane landed.
a) 6:00 am b) 6:40am c)7:40 d)7:00 e)8:00 (Hint : Every 1 deg longitude is equal to 4 minutes . If west to east add time
else subtract time) ANS: (E) 8:00
48. Find the highest prime number that can be stored in an 8bit computer.
49. Which of the following set of numbers has the highest Standard deviation?
1,0,1,0,1,0
-1,-1,-1,-1,-1,-1
1,1,1,1,1,1
1,1,0,-1,0,-1
50. Match the following:
A superset of

1. Male - Boy --->a. A type of

2. Square - Polygon ---> b. A part of

3. Roof - Building --->c. Not a type of

4. Mushroom - Vegetables ---> d.

Ans: 1- d, 2- a, 3- b, 4- c

51. Match the following.


1. brother - sister
---> a. Part of
2. Alsatian - dog --->
b. Sibling
3. sentence - paragraph --->
c. Type of
4. car - steering
---> d. Not a type
of
Ans. 1-b, 2-c, 3-a, 4-d
1. Two pencils costs 8 cents, then 5 pencils cost how much (Ans:20 cents).
2. A work is done by the people in 24 min. one of them can do this work a lonely in 40 min. how much time required to do
the same work for the second person. (ans:60 min.)
3. A car is filled with four and half gallons of oil for fullround trip. fuel is taken 1/4 gallons mor3 in going than coming. what
is the fiel consumed in coming up? (2 gallons)
4. low temperature at the night in a city is 1/3 more than 1/2 hinge as higher temperature in a day. sum of the low temp
and higherst temp is 100C. then what is the low temperature (40 C)
5. A person who decided to go weekend trip should not exceed 8hoursdriving in a day Average speed of forward journy is 40
mph. due to traffic in sundays, the return journey average speed is 30 mph. how far he can select a picnic spot (120 miles).
6. A sales person multiplied a number and get the answer is 3,instead of that number divided by 3. what is th answer he
actuallyhas to get ? (1/3).
7. A ship started from port and moving with I mph and another shipstarted from L and moving with H mph. At which place
these two ships meet ? ( Ans is between I and J and close to J)
!_____!_____!_____!_____!_____!_____!
port G H I J K L
8. A building with hight D ft shadow upto G A neighbour building
with what height shadow C ft is (B ft.)
!_____!_____!_____!_____!_____!_____!_____!
ABCDEFGH
Edited by: Riyaz Ali Khan Mayana

B. Tech .(CSE), D. E. C. E.

Email: riyaz666@gmail.com

26

9. A person was fined for exceeding the speed limit by 10 mph.Anotherperson was also fined for exceeding the same speed
limit by twicethe same. If the second person was travelling at a speed of 35mph. find the speed limit (15 mph)
10. A bus started from bustand at 8.00a m and after 30 min stayingat destination, it returned back to the bustand. the
destination is 27 miles from the bustand. the speed of the bus 50 percent fastspeed.at what time it retur4ns to the bustand
(11.00)
11.in a mixture, R is 2 parts, S is 1 part. in order to make S to25% of the mixture, howmuch R is to be added ( one part).
12. wind flows 160 miles in 330 min, for 80 miles how much timerequired.
13. with 4/5 full tank vehicle travels 12 miles, with 1/3 full tank how much distance travels ( 5 miles).
14. two trees are there. one grows at 3/5 of the other. in 4 years, total growth of trees is 8 ft. what growth will smaller tree
willhave in 2 years. (<2ft)
15. A storm will move with a velocity of towords the center inhours. At the same rate how much far will it move in hrs.
(but Ans is 8/3 or 2 2/3).

Papers From Campus Recruitment at Calicut REC 1997


1. If two pencils cost 8 cents, then how much do 5 pencils cost?Ans. 20 cents
2. Some work is done by two people in 24 minutes. One of them can do this work alone in 40 minutes. How much time does
the second person take to do the same work ? Ans. 60 minutes
3. A car is filled with four and half gallons of fuel for a round trip.If the amount of fuel taken while going is 1/4 more than the
amount taken for coming, what is the amount of fuel consumed while coming back? Ans.2 gallons
4. The lowest temperature in the night in a city A is 1/3 more than 1/2 the highest during the day. Sum of the lowest
temperature and the highest temperature is 100 degrees. Then what is the low temp? Ans.40 degrees
5. Javagal, who decided to go to weekened trip should not exceed 8 hours driving in a day. The average speed of forward
journey is 40 miles/hr.Due to traffic on sundays, the return journey's average speed is 30 m/h. How far he can select a picnic
spot?
a) 120 miles b) between 120 and 140 miles c) 160 miles
Ans. 120 miles
6. A salesperson by mistake multiplied a number and got the answer as 3, instead of dividing the number by 3.What is the
answer he should have actually got? Ans. 3
7. A building with height D shadow upto G. What is the height of a neighbouring building with a shadow of C feet. Ans.
(C*D)/G
8. A person was fined for exceeding the speed limit by 10 mph. Another person was also fined for exceeding the same speed
limit by twice the same. If the second person was travelling at a speed of 35 mph, find the speed limit. Ans. 15 mph
9. A bus started from bustand at 8.00am, and after staying for 30 minutes at a destination, it returned back to the busstand.
The destination is 27 miles from the busstand. The speed of the bus is 18mph. During the return journey bus travels with
50% faster speed.At what time does it return to the busstand? Ans. 11.00am
10. In a mixture, R is 2 parts and S is 1 part. In order to make S to 25% of the mixture, how much of R is to be added?
Ans.One part of R
11. Wind flows 160 miles in 330 min, for travelling 80 miles how much time does it require?Ans. 2 hrs 45 mins
12. With a 4/5 full tank a vehicle can travel 12 miles, how far can it travel with a 1/3 full tank Ans. 5 miles
13. There are two trees in a lawn. One grows at a rate 3/5 of the other in 4 years. If the total growth of trees is 8 ft. What is
the height of the smaller tree after 2 years Ans. 1 1/2 feet
14. Refer to the figure below.A ship started from P and moves at a speed of I miles per hour and another ship starts from L
and moving with H miles per hour simultaneously.Where do the two ships meet?
Edited by: Riyaz Ali Khan Mayana

B. Tech .(CSE), D. E. C. E.

Email: riyaz666@gmail.com

27

||---g---||---h---||---i---||---j---||---k---||---l---||
PG H I J K L are the various stops in between denoted by || . The values g, h, i, j, k, l denote the distance between the ports.
Ans. Between I and J, closer to J
15. If A is travelling at 72 km per hour on a highway. B is travelling at a speed of 25 meters per second on a highway. What is
the difference in their speeds in m/sec. Ans. 1 m/sec
IV SECTION
1. There are 150 weights .Some are 1 kg weights and some are 2 kg weights. The sum of the weights is 260.What is the
number of 1kg weights? Ans. 40
2. A is driving on a highway when the police fines him for overspeeding and exceeding the limit by 10 km/hr.At the same time
B is fined for overspeeding by twice the amount by which A exceeded the limit.If he was driving at 35 km/hr what is the
speed limit for the road? Ans. 15 kmph
3. A moves 3 kms east from his starting point . He then travels 5 kms north. From that point he moves 8 kms to the
east.How far is A from his starting point? Ans. 13 kms
4. A car travels 12 kms with a 4/5th filled tank.How far will the car travel with 1/3 filled tank? Ans. 5 kms
5. The sum of the digits of a two digit number is 8. When 18 is added to the number, the digits are reversed. Find the
number?
Ans. 35
6. The cost of one pencil, two pens and four erasers is Rs.22 while the cost of five pencils, four pens and two erasers is
Rs.32.How much will three pencils, three pens and three erasers cost? Ans. 27
7. Fathers age is 5 times his son's age. 4 years back the father was 9 times older than son.Find the fathers' present age. Ans.
40 years
8. What number should be added to or subtracted from each term of the ratio 17 : 24 so that it becomes equal to 1 : 2.
Ans. 10 should be subtracted
9. What is the 12th term of the series 2, 5, 8, .... Ans. 35
10. If 20 men take 15 days to to complete a job, in how many days can 25 men finish that work? Ans. 12 days
11. In a fraction, if 1 is added to both the numerator at the denominator, the fraction becomes 1/2. If numerator is
subtracted from the denominator, the fraction becomes 3/4. Find the fraction. Ans. 3/7
12. If Rs.1260 is divided between between A, B and C in the ratio 2:3:4, what is C's share? Ans. Rs. 560
13. A shopkeeper bought a watch for Rs.400 and sold it for Rs.500.What is his profit percentage? Ans. 25%
14. What percent of 60 is 12?Ans. 20%
15. Hansie made the following amounts in seven games of cricket in India: Rs.10, Rs.15, Rs.21, Rs.12, Rs.18, Rs.19 and
Rs.17(all figures in crores of course).Find his average earnings. Ans. Rs.16 crore
Quantitative:
1. 3 angles or 3 sides r given.Which will form a triangle?
2. units of basic quantities :
1. (energy * time * time )/(mass * dist) = distance
2. (momentum * velocity)/(force * time) = velocity
3.& is for doubling the value % is for change of sign then what is the value
5-&%&5 Ans-30 (Check)
3. 58,27,12,x,2,1. Find x.
4. R-rounding off, M-modulus, T-truncate

M(893,10)+r( )+t( ) is asked

5.vertices edges and surfaces of a cube Ans-8,12,6


6.Sums on Recursive functions
7.Questions on General computer awareness
Pick the odd one..
1.http 2.arp 3.snmp 4.sap Ans-sap
1.linux 2.windows NT 3.sql server 4.Unix Ans-Sql server Another..ans-Smtp
Ans-MVS
8. Which of the following is a singular matrix. (Determinant must be zero)
9. Aeroplane is flying at a particular angle and latitude,after some time another latitude is given..(8 hrs later), u r asked to
Edited by: Riyaz Ali Khan Mayana

B. Tech .(CSE), D. E. C. E.

Email: riyaz666@gmail.com

28

find the local time of the place.


10.a series of letters are given
how many Ws r followed by F and preceded by T.
11. 7,9,13,_,27,37. Ans-19
12.SURFW Code is translated as SHEET..these kinda ques r there.. Freshersworld.com
13.194 base 10 = ____ base 5 (1234)
14.Largest prime no. in a 6 bit,8 bit (Ans 127),9 bit microprocessor
15.Venn Diagram kinda ques.
Some know English, some French,some Germanhow many know two languages..
16.Bar Diagram, Pie Chart (similar to Data interpretation)
17.Code Interchanging, A word is given. Letters are reversed..u r asked to find the nth letter from right or left.
Eg. DESTABILIZATION Ans-T
18. Sums on logarithms, e power x curves.
19.n=68 x 12 x 51
Which of the follg is not an integer Ans- n/122
20.Which is a/not a power of 2 or 3.
Power of 4 Ans-4096
21. A-- 1 0 10 10 (Not exact values)
SECTION I
1. If VXUPLVH is written as SURMISE, what is SHDVD ?
Ans. PEASA (hint: in the first
word, the alphabets of the jumbled one is three alphabets after the corresponding alphabet in the word SURMISE. S = V-3,
similarly find the one for SHDVD)
2. If DDMUQZM is coded as CENTRAL then RBDJK can be coded as --------Ans. QCEIL (hint: Write both the jumbled and the coded word as a table, find the relation between the
corresponding words, i.e C= D-1, N=M+1 & so on
3. In the word ECONOMETRICS, if the first and second , third and forth ,forth and fifth, fifth and sixth words are
interchanged up to the last letter, what would be the tenth letter from right?
Ans. word is CENOMOTEIRSC tenth word is R
4. Find the result of the following __expression if, M denotes modulus operation, R denotes round-off, T denotes truncation:
M(373,5)+R(3.4)+T(7.7)+R(5.8)

Ans. 19

5. What is the largest prime number that can be stored in an 8-bit memory?
6. Find the physical quantity in units from the equation: (Force*Distance)/(Velocity*Velocity)

Ans. Ns2/m

7. Find the value of @@+25-++@16, where @ denotes "square" and + denotes "square root".

Ans: 621

8. If f(0)=1 and f(n)= f(n-1)*n, find the value of f(4).

Ans: 24

9. Convert the decimal number 310 to the base 6.


10. Find the missing number in the series: 2,

5,

Ans: 1234
__ ,

19 ,

37,

75

Ans: 9

11. In a two-dimensional array, X(9,7), with each element occupying 4 bytes of memory, with the address of the first element
X(1,1) is 3000, find the address of X(8,5).

Ans.

12. Find the fourth row, having the bit pattern as an integer in an 8-bit computer, and express the answer in its Decimal
value.
A

0 0 0 0 1 1 1 1

0 0 1 1 0 0 1 1

0 1 0 1 0 1 0 1

(AU(B-C))

Ans. 29

13. Complete the series 2, 7, 24, 77,__ (hint: 2*12= 24, 7*11= 77, therefore 24*10= 240)

Ans: 240

14. Consider the following diagram for answering the following questions:
A. Find the difference between people playing cricket and tennis alone.
Edited by: Riyaz Ali Khan Mayana

B. Tech .(CSE), D. E. C. E.

Ans: 4
Email: riyaz666@gmail.com

29

B. Find the percentage of people playing hockey to that playing both hockey and cricket.

Ans:

C. Find the percentage of people playing all the games to the total number of players.

Ans: 6%

15. One more question of the same type (Same type of diagram; of course in a different set)
1. How many more or less speak English thanFrench?
2. What % people speak all the three languages?
3. What % people speak German but not English?
{In another set cricket, hockey and tennis are changed with the name of some computer languages, such as
Java, Cobol, Fortran (may be some other name)}
16. Select the odd one out

a. Oracle

17. Select the odd one out


18. Select the odd man out.

b.Linux

c. Ingress

a. SMTP

b. WAP

c. SAP

a. Java

b.Lisp

c. Smalltalk

d. DB2
d. ARP
d. Eiffel

19. Which of the following are orthogonal pairs?


a. 3i+2j

b. i+j

c. 2i-3j

d. -7i+j

20. Number of faces, vertices and edges of a cube


a. 12,8,6

b. 4,6,8

c. 6,8,12

d. 6,12,8

21. Given a Bar Chart showing the sales of a company. (In Figure) The sales in years as shown in the figure are (in crores)
1998-1999 - 130, 1997-1998 - 90, 1996-1997 - 90, 1995-1996 - 70
1. The highest growth rate was for the year

Ans. 1998-1999

2. The net increase in sales of the company in the year span of 1995-1999

Ans. 60 crores.

3. The lowest growth rate was for the year

Ans. 1997

22. Find the value of the decimal number to the base 7.

Ans. 1436.

23. Complete the series:5,6,7,8,10,11,14,__.

Ans. 15

24. If the vertex (5,7) is placed in the memory. First vertex (1,1) s address is 1245 and then address of (5,7) is -----25. In which of the system, decimal number 384 is equal to 1234?

Ans.

26. A man, a woman, and a child can do a piece of work in 6 days. Man only can do it in 24 days. Woman can do it in 16 days
and in how many days child can do the same work?

Ans.

27. In Madras, temperature at noon varies according to -t^2/2 + 8t + 3, where t is elapsed time. Find how much
temperature more or less in 4pm to 9pm.

Ans.

28. The size of the bucket is N kb. The bucket fills at the rate of 0.1 kb per millisecond. A programmer sends a program to
receiver. There it waits for 10 milliseconds. And response will be back to programmer in 20 milliseconds. How much time the
program takes to get a response back to the programmer, after it is sent?

Ans.

29. The size of a program is N. And the memory occupied by the program is given by M = square root of 100N. If the size of
the program is increased by 1% then how much memory now occupied ?

Ans.

30. A power unit is there by the bank of the river of 750 meters width. A cable is made from power unit to power a plant
opposite to that of the river and 1500mts away from the power unit. The cost of the cable below water is Rs. 15/- per meter
and cost of cable on the bank is Rs.12/- per meter. Find the total of laying the cable.

Ans. Rs. 22,500 (hint: the

plant is on the other side of the plant i.e. it is not on the same side as the river)
{There are two questions, both showing a curve. In the first one, you have to identify the curve. In the second one you have
to write the equation of the curve. In}
ANALYTICAL:
i am sending the questions.these r the models.the same type may be asked but with different numbers.if u solve all of these
u can easily solve all the problems in the main paper
1)

Bhanu spends 30% of his income on petrol on scooter. of the remaining on house rent and the balance on food. If

he spends Rs.300 on petrol then what is the expenditure on house rent?


a)

Rs.525

b)

Rs.1000

Edited by: Riyaz Ali Khan Mayana

c)

B. Tech .(CSE), D. E. C. E.

Rs.675

d)

Rs.175

Email: riyaz666@gmail.com

A
2)

30

If the numerator of a fraction is increased by 25% and denominator decreased by 20%, the new value is 5/4. What is

the original value?


a)

3/5

b)

4/5

c)

7/8

d)

3/7

3)

The length of a rectangle is increased by 60%. By what % would the width have to be decreased to maintain the

same area?
a)

30%

b)

60%

4)

The value of + 5 / 36 + 7 / 144 + .+17 / 5184 + 19 / 8100 is

a)

0.99

5)

A sporting goods store ordered an equal number of white and yellow balls. The tennis ball company delivered 45

b)

c)

0.98

75%

c)

d)

0.95

37.5%

d)

None of these

extra white balls, making the ratio of white balls to yellow balls 1/5 : 1/6. How many white tennis balls did the store originally
order for?
a)

450

b)

270

c)

225

d)

None of these

6)

A student's grade in a course is determined by 6 quizzes and one examination. If the examination counts thrice as

much as each of the quizzes, what fraction of final grade is determined by the examination?
a)

1/6

b)

1/5

c)

1/3

d)

7)

A sum of money is divided among A, B and C such that for each rupee A gets, B gets 65paise and C gets 35paise. If

C's share is Rs.560, the sum is


a)
8)

2400

b)

2800

c)

3200

d)

3800

Joe's father will be twice his age 6 years from now. His mother was twice his age 2 years before. If Joe will be 24

two years from now, what is the difference between his father's and mother's age?
a)

b)

c)

d)

10

9)

A traveler walks a certain distance. Had he gone half a kilometer an hour faster \, he would have walked it in 4/5 of

the time, and had he gone half a Kilometer an hour slower, he would have walked 2 hr longer. What is the distance?
a)

10 Km

b)

15 Km

c)

20 Km

d)

Data Insufficient

10)

Two oranges, 3 bananas and 4 apples cost Rs.15. 3 oranges, 2 bananas and 1 apple cost Rs.10. I bought 3 oranges,

3 bananas and 3 apples. How much did I pay?


a)

10

b)

c)

15

d)

Cannot be determined

11)

A report consists of 20 sheets each of 55 lines and each such line consists of 65 characters. This report is retyped

into sheets each of 65 lines such that each line consists of 70 characters. The % reduction in the number of sheets is closest
to
a)

20

b)

c)

30

d)

35

12)

A ship leaves on a long voyage. When it is 18 miles from the shore, a seaplane, whose speed is 10 times that of the

ship is sent to deliver mail. How far from the shore does the seaplane catch upo with the ship?
a)

24 miles

b)

Edited by: Riyaz Ali Khan Mayana

25 miles c)
B. Tech .(CSE), D. E. C. E.

22miles

d)

20 miles
Email: riyaz666@gmail.com

13)

31

Anand finishes a work in 7 days, Bittu finishes the same job in 8 days and Chandu in 6 days. They take turns to

finish the work. Anand on the first day, Bittu on the second and Chandu on the third day and then Anand again and so on. On
which day will the work get over?
a)

3rd

b)

6th

c)

9th

d)

7th

14)

3 men finish painting a wall in 8 days. Four boys do the same job in 7 days. In how many days will 2 men and 2 boys

working together paint two such walls of the same size?


a)

6 6/13 days

b)

3 3/13 days

c)

9 2/5 days

d)

12 12/13 days

15)

There are 5 distinct pairs of white socks and 5 pairs of black socks in a cupboard. In the dark, how many socks do I

have to pull out to ensure that I have at least 1 correct pair of white socks?
a)

b)

11

c)

12

d)

16

16)

In a circular race track of length 100 m, three persons A, B and C start together. A and B start in the same direction

at speeds of 10 m/s and 8 m/s respectively. While C runs in the opposite at 15 m/s. When will all the three meet for the first
time on the after the start?
a)

After 4 s

b)

After 50 s

c)

After 100 s

d)

After 200 s

17)

If the distance traveled (s) in time (t) by a partile is given by the formula s = 1+ 2t+3t2+4t3 , then what is the

distance travelled in the 4th second of its motion?


a)

141m

b)

171m

c)

243m

d)

313m

18)

There is a circular pizza with negligible thickness that is cut into 'x' pieces by 4 straight line cuts. What is the

maximum and minimum value of 'x' respectively?


a)

12,6

b)

11,6

c)

12,5

d)

11,5

19)

When of a unit's digit is added to the ten's digit of a two number, the sum of the digits becomes 10. If of the

ten's digit added to the unit's digit, then the sum of the digits is 1 less than the previous. Find the number.
a)

94

20)

LCM of x and y is 36 and HCF of x and y is 4. If x = 4 then which of the following is definitely not true?

a) Y is divisible by 3
21)

b)

84

b) Y is divisible by 6

c)

48

d)

c)Y is divisible by 9

d)

88

X+Y is divisible by

Amal bought 5 pens, 7 pencils and 4 erasers. Rajan bought 6 pens, 8 erasers and 14 pencils for an amount which

was half more than what Amal had paid. What % of the total amount paid by Amal was paid for pens?
a)
22)

37.5%

b)

62.5%

c)

50%

d)

None of these

A non stop bus to Amritsar overtakes an auto also moving towards Amritsar at 10 am. The bus reaches Amritsar at

12.30 pm and starts on the return journey after 1 hr. On the way back it meets the auto at 2 pm. At what time the auto will
reach Amritsar?
a)

2.30pm

b)

3.00pm

c)

3.15pm d)

3.30pm

23)

The total expense of a boarding house are partly fixed and partly variable with the number of boarders. The charge is

Rs.70 per head when there are 25 boarders and Rs.60 when there are 50 boarders. Find the charge per head when there are
100 boarders.
a)

65

b)

24)

How many positive integer solutions does the equation 2x+3y = 100 have?

a)

50

b)

Edited by: Riyaz Ali Khan Mayana

55

33

B. Tech .(CSE), D. E. C. E.

c)

c)

50

16

d)

d)

45

35

Email: riyaz666@gmail.com

A
25)

32

A person had to multiply two numbers. Instead of multiplying by 35, he multiplied by 53and the product went up by

540. What was the raised product?


a)

780

1.If g (0)=g (1)=1

b)

1040

c)

1590

d)

1720

And g (n)= g (n-1) + g (n -2) find g (6);

2.A plane moves from 9?N40?E to 9?N40?W. If the plane starts at 10 am and takes 8 hours to reach the destination, find the
local arrival time.
3. If log 0.317= and log 0.318=. Then find the value of log 0.319.
4.You will be given the bit position values for A, B and C and using the relation (A?B) u C you have to construct the truth
table. Then find the corresponding decimal number and choose the right option.
5.Complete the sequence 9,10,11,13,15, __, 21,28.
6.In a certain format TUBUJPO is coded as STATION. The code of which string is FILTER?
7. What is the code formed by reversing the First and second letters, the third and fourth letters and son on of the string
SIMULTANEOUSLY?
8.The base 5 representation of the decimal number 2048 is _____.
9.Which is the largest prime number that can be stored in a 9-bit register?
10.Find the physical quantity represented by MOMENTUM *VELOCITY] / [LENGTH * ACCELERATION]?
11.A can do a piece of work in 20 days, which B can do in 12 days. In 9 days B does of the work. How many days will A
take to finish the remaining work?
1. In a two-dimensional array, X (9, 7), with each element occupying 4 bytes of memory, with the address of the first
element X (1, 1) is 3000, find the address of X (8, 5).
ANS.: 3212
2. In the word ORGANISATIONAL, if the first and second, third and forth, forth and fifth, fifth and sixth words are
interchanged up to the last letter, what would be the tenth letter from right?
ANS.: I(ROANISATIONALG)
2E. In the word ORGANISATIONAL, if the first and second, third and forth, fifth and sixth words are interchanged up to the
last letter, what would be the tenth letter from right?
ANS.: I(ROAGINASITNOLA)
3. What is the largest prime number that can be stored in an 8-bit memory?

ANS.:127

4. Select the odd one out. a. Java b. Lisp c. Smalltalk d.Eiffel.

ANS.: LISP

5. Select the odd one out a. SMTP b. WAP c. SAP d. ARP

ANS.: SAP

6. Select the odd one out a. Oracle b. Linux c. Ingress d. DB2

ANS.:LINUX

7. Select the odd one out a. WAP b. HTTP c. BAAN d. ARP

ANS.:BAAN

8. Select the odd one out a. LINUX b. UNIX c.SOLARIS d. SQL SERVER

ANS.:SQL SERVER

9. Select the odd one out a. SQL b. DB2 c.SYBASE d. HTTP

ANS.:HTTP

Edited by: Riyaz Ali Khan Mayana

B. Tech .(CSE), D. E. C. E.

Email: riyaz666@gmail.com

33

10. The size of a program is N. And the memory occupied by the program is given by M = square root of 100N. If the size of
the program is increased by 1% then how much memory now occupied?
11. A man, a woman, and a child can do a piece of work in 6 days. Man only can do it in 24 days. Woman can do it in 16 days
and in how many days child can do the same work?

ANS.:16

12. In which of the system, decimal number 194 is equal to 1234?

ANS.:5

13. Find the value of the 678 to the base 7.

ANS.:1656

14. Number of faces, vertices and edges of a cube

ANS.:6,8,12

15. Complete the series 2, 7, 24, 77,__

ANS.:238

16. Find the value of @@+25-++@1..., where @ denotes "square" and + denotes "square root". ANS.:121
17. Find the result of the following _expression if, M denotes modulus operation, R denotes round-off, T denotes truncation:
M(373,5)+R(3.4)+T(7.7)+R(5.8) ANS.:19
18. If TAFJHH is coded as RBEKGI then RBDJK can be coded as ---------

ANS.:PCCKJ

19. G(0)=-1, G(1)=1, G(N)=G(N-1) - G(N-2), G(5)= ?

ANS.:-2

20. What is the max possible 3 digit prime number?

ANS.:

21. A power unit is there by the bank of the river of 750 meters width. A cable is made from power unit to power a plant
opposite to that of the river and 1500mts away from the power unit. The cost of the
cable below water is Rs. 15/- per meter and cost of cable on the bank is Rs.12/- per meter. Find the total of laying the cable.
ANS:20250
22. The size of a program is N. And the memory occupied by the program is given by M = square root of 100N. If the size of
the program is increased by 1% then how much memory now occupied?
23. In Madras, temperature at noon varies according to -t^2/2 + 8t + 3, where t is elapsed time. Find how much
temperature more or less in 4pm to 9pm.

ANS.: 385.8(DB)

24. The size of the bucket is N kb. The bucket fills at the rate of 0.1 kb per millisecond. A programmer sends a program to
receiver. There it waits for 10 milliseconds. And response will be back to programmer in 20 milliseconds. How much time the
program takes to get a response back to the programmer, after it is sent?

ANS.: 30MILISECOND

25. A man, a woman, and a child can do a piece of work in 6 days. Man only can do it in 24 days. Woman can do it in 16
days and in how many days child can do the same work?
26. If the vertex (5,7) is placed in the memory. First vertex (1,1) ?s address is 1245 and then address of (5,7) is ---------27. Which of the following are orthogonal pairs?

a. 3i+2j b. i+j

c. 2i-3j d. -7i+j

28. If VXUPLVH is written as SURMISE, what is SHDVD?

ANS.: (A)& (C).

ANS.: PEASA

29. If A, B and C are the mechanisms used separately to reduce the wastage of fuel by 30%, 20% and 10%. What will be the
fuel economy if they were used combined.

ANS.: 20%

30. What is the power of 2? a. 2068 b.2048 c.2668

ANS.: (B). 2048

31. Complete the series. 3, 8, --, 24, --, 48, 63

ANS.: 15,35

32. Complete the series. 4, -5, 11, -14, 22, ---

ANS.: -27

33. A, B and C are 8 bit no?s. They are as follows:


A11011011

B01111010

Hint :

ANS.: 0 1 1 1 1 1 1 1 (DB)

A-B is {A} - {A n B}

C01101101

Find ( (A-B) u C )=?

A Flight takes off at 2 A.M from northeast direction and travels for 11 hours to reach the destination which is in north west
direction.Given the latitude and longitude of source and destination. Find the
local time of destination when the flight reaches there?

ANS.: 1:00 P.M

35. A can copy 50 papers in 10 hours while both A & B can copy 70 papers in 10 hours. Then for how many hours required
for B to copy 26 papers?

ANS.: 13

36. A is twice efficient than B. A and B can both work together to complete a work in 7 days. Then find in how many days A
alone can complete the work?

ANS.: 10.5 DAYS(11)

37. A finish the work in 10 days. B is 60% efficient than A. So hoW days does B take to finish the work?ANS. : 4DAYS.
38. A finishes the work in 10 days & B in 8 days individually. If A works for only 6 days then how many days should B work to
complete A?s work?

Edited by: Riyaz Ali Khan Mayana

ANS. : 3.2 DAYS(4)

B. Tech .(CSE), D. E. C. E.

Email: riyaz666@gmail.com

34

39. Given the length of the 3 sides of a triangle. Find the one that is impossible? (HINT : sum of smaller 2 sides is greater
than the other one which is larger)
40. Find the singularity matrix from a given set of matrices?(Hint det(A)==0)
41. A 2D array is declared as A[9,7] and each element requires 2 byte.If A[ 1,1] is stored in 3000. Find the memory of A[8,5]
?

ANS.: 3106.

42. Sum of slopes of 2 perpendicular st. lines is given. Find the pair of lines from the given set of options which satisfy the
above condition?
43. (a) 2+3i (b)1+i (c) 3-2i (d) 1-7i .Find which of the above is orthogonal.

ANS. : (A) & (C).

44. (Momentum*Velocity)/(Acceleration * distance ) find units.

ANS.:MASS

45. The number 362 in decimal system is given by (1362)x in the X system of numbers find the value of X a}5 b) 6 c) 7 d) 8
e) 9
46. Given $ meAns. Tripling and % meAns. change of sign then find the value of $%$6-%$%6

ANS. : -72

47. My flight takes of at 2am from a place at 18N 10E and landed 10 Hrs later at a place with coordinates 36N70W. What is
the local time when my plane landed.
a) 6:00 am b) 6:40am c)7:40 d)7:00
e)8:00 (Hint : Every 1 deg longitude is equal to 4 minutes . If west to east add time else subtract time)

ANS.: (E) 8:00

48. Find the highest prime number that can be stored in an 8bit computer.
49. Which of the following set of numbers has the highest Standard deviation?
1,0,1,0,1,0
-1,-1,-1,-1,-1,-1
1,1,1,1,1,1
1,1,0,-1,0,-1
50. Match the following:
1. Male - Boy --->
a. A type of
2. Square - Polygon --->
b. A part of
3. Roof - Building --->
c. Not a
type of
4. Mushroom - Vegetables ---> d.
A superset of
Ans.: 1- d, 2- a, 3- b, 4- c
51. Match the following.
1. brother - sister
---> a. Part of
2. Alsatian - dog --->
b. Sibling
3. sentence - paragraph --->
c. Type of
4. car - steering
---> d. Not a type
of
Ans.. 1-b, 2-c, 3-a, 4-d
1. Two pencils costs 8 cents, then 5 pencils cost how much

(Ans.:20 cents).

2. A work is done by the people in 24 min. one of them can do this work a lonely in 40 min. how much time required to do
the same work for the second person.

Edited by: Riyaz Ali Khan Mayana

B. Tech .(CSE), D. E. C. E.

(Ans.:60 min.)

Email: riyaz666@gmail.com

35

3. A car is filled with four and half gallons of oil for full round trip. fuel is taken 1/4 gallons mor3 in going than coming. what
is the fiel consumed in coming up?

(2 gallons)

4. low temperature at the night in a city is 1/3 more than hinge as higher temperature in a day. sum of the low temp and
higherst temp is 100C. then what is the low temperature

(40 C)

5. A person who decided to go weekend trip should not exceed 8 hours driving in a day Average speed of forward journy is
40 mph.due to traffic in sundays, the return journey average speed is 30 mph.how far he can select a picnic spot (120miles).
6. A sales person multiplied a number and get the Ans.wer is 3, instead of that number divided by 3. what is th Ans.wer he
actually has to get ?

(1/3).

7. A ship started from port and moving with I mph and another ship started from L and moving with H mph. At which place
these two ships meet ?

( Ans. is between I and J and close to J)

!_____!_____!_____!_____!_____!_____!
port G H I J K L
8. A building with hight D ft shadow upto G A neighbour building with what height shadow C ft is (B ft.)
!_____!_____!_____!_____!_____!_____!_____!
ABCDEFGH
9. A person was fined for exceeding the speed limit by 10 mph.Another person was also fined for exceeding the same speed
limit by twice the same. If the second person was travelling at a speed of 35 mph. find the speed limit

(15 mph)

10. A bus started from bustand at 8.00a m and after 30 min staying at destination, it returned back to the bustand. the
destination is 27 miles from the bustand. the speed of the bus 50 percent fast speed. at what time it retur4ns to the
bustand

(11.00)

11.in a mixture, R is 2 parts, S is 1 part. in order to make S to 25% of the mixture, howmuch R is to be added ( one part).
12. wind flows 160 miles in 330 min, for 80 miles how much time required.
13. with 4/5 full tank vehicle travels 12 miles, with 1/3 full tank how much distance travels

( 5 miles).

14. two trees are there. one grows at 3/5 of the other. in 4 years, total growth of trees is 8 ft. what growth will smaller tree
will have in 2 years.

(<2ft)

15. A storm will move with a velocity of towords the center in hours. At the same rate how much far will it move in hrs.
(but Ans is 8/3 or 2 2/3).
1. If two pencils cost 8 cents, then how much do 5 pencils cost?

Ans. 20 cents

2. Some work is done by two people in 24 minutes. One of them can do this work alone in 40 minutes. How much time does
the second person take to do the same work ?

Ans.. 60 minutes

3. A car is filled with four and half gallons of fuel for a round trip.If the amount of fuel taken while going is 1/4 more than the
amount taken for coming, what is the amount of fuel consumed while coming back?

Ans..2 gallons

4. The lowest temperature in the night in a city A is 1/3 more than 1/2 the highest during the day. Sum of the lowest
temperature and the highest temperature is 100 degrees. Then what is the low temp?

Ans..40 degrees

5. Javagal, who decided to go to weekened trip should not exceed 8 hours driving in a day. The average speed of forward
journey is 40 miles/hr.Due to traffic on sundays, the return journey's average speed is 30 m/h. How far he can select a picnic
spot?

a) 120 miles

b) between 120 and 140 miles

c) 160 miles

Ans.. 120 miles

6. A salesperson by mistake multiplied a number and got the Ans.wer as 3, instead of dividing the number by 3.What is the
Ans.wer he should have actually got?

Ans.. 3

7. A building with height D shadow upto G. What is the height of a neighbouring building with a shadow of C feet.
Ans.. (C*D)/G
8. A person was fined for exceeding the speed limit by 10 mph. Another person was also fined for exceeding the same speed
limit by twice the same. If the second person was travelling at a speed of 35 mph, find the speed limit.

Edited by: Riyaz Ali Khan Mayana

B. Tech .(CSE), D. E. C. E.

Ans.. 15 mph

Email: riyaz666@gmail.com

36

9. A bus started from bustand at 8.00am, and after staying for 30 minutes at a destination, it returned back to the busstand.
The destination is 27 miles from the busstand. The speed of the bus is 18mph. During the return journey bus travels with
50% faster speed.At what time does it return to the busstand?

Ans.. 11.00am

10. In a mixture, R is 2 parts and S is 1 part. In order to make S to 25% of the mixture, how much of R is to be added?
Ans..One part of R

11. Wind flows 160 miles in 330 min, for travelling 80 miles how much time does it require?

Ans. 2 hrs 45 mins

12. With a 4/5 full tank a vehicle can travel 12 miles, how far can it travel with a 1/3 full tank
Ans.. 5 miles
13. There are two trees in a lawn. One grows at a rate 3/5 of the other in 4 years. If the total growth of trees is 8 ft. What is
the height of the smaller tree after 2 years
Ans.. 1 1/2 feet
14. Refer to the figure below.A ship started from P and moves at a speed of I miles per hour and another ship starts from L
and moving with H miles per hour simultaneously.Where do the two ships meet?
||---g---||---h---||---i---||---j---||---k---||---l---||
PG H I J K L are the various stops in between denoted by || . The values g, h, i, j, k, l denote the distance between the ports.
Ans.. Between I and J, closer to J
15. If A is travelling at 72 km per hour on a highway. B is travelling at a speed of 25 meters per second on a highway. What is
the difference in their speeds in m/sec.

Ans.. 1 m/sec

1. There are 150 weights .Some are 1 kg weights and some are 2 kg weights. The sum of the weights is 260.What is the
number of 1kg weights?

Ans.. 40

2. A is driving on a highway when the police fines him for overspeeding and exceeding the limit by 10 km/hr.At the same time
B is fined for overspeeding by twice the amount by which A exceeded the limit.If he was driving at 35 km/hr what is the
speed limit for the road?

Ans.. 15 kmph

3. A moves 3 kms east from his starting point . He then travels 5 kms north. From that point he moves 8 kms to the
east.How far is A from his starting point?

Ans.. 13 kms

4. A car travels 12 kms with a 4/5th filled tank.How far will the car travel with 1/3 filled tank?

Ans.. 5 kms

5. The sum of the digits of a two digit number is 8. When 18 is added to the number, the digits are reversed. Find the
number?

Ans.. 35

6. The cost of one pencil, two pens and four erasers is Rs.22 while the cost of five pencils, four pens and two erasers is
Rs.32.How much will three pencils, three pens and three erasers cost?

Ans.. 27

7. Fathers age is 5 times his son's age. 4 years back the father was 9 times older than son.Find the fathers' present age.
Ans.. 40 years
8. What number should be added to or subtracted from each term of the ratio 17 : 24 so that it becomes equal to 1 : 2.
Ans.. 10 should be subtracted
9. What is the 12th term of the series 2, 5, 8, ....

Ans.. 35

10. If 20 men take 15 days to to complete a job, in how many days can 25 men finish that work?

Ans.. 12 days

11. In a fraction, if 1 is added to both the numerator at the denominator, the fraction becomes 1/2. If numerator is
subtracted from the denominator, the fraction becomes 3/4. Find the fraction.

Ans.. 3/7

12. If Rs.1260 is divided between between A, B and C in the ratio 2:3:4, what is C's share?
Edited by: Riyaz Ali Khan Mayana

B. Tech .(CSE), D. E. C. E.

Ans.. Rs. 560


Email: riyaz666@gmail.com

37

13. A shopkeeper bought a watch for Rs.400 and sold it for Rs.500.What is his profit percentage? Ans.. 25%
14. What percent of 60 is 12?

Ans.. 20%

15. HAns.ie made the following amounts in seven games of cricket in India: Rs.10, Rs.15, Rs.21, Rs.12, Rs.18, Rs.19 and
Rs.17(all figures in crores of course).Find his average earnings.

Ans.. Rs.16 crore

Quantitative:
1. 3 angles or 3 sides r given.Which will form a triangle?
2. units of basic quantities :
1. (energy * time * time )/(mass * dist) = distance Freshersworld.com
2. (momentum * velocity)/(force * time) = velocity
3.& is for doubling the value % is for change of sign then what is the value 5-&%&5 Ans.-30 (Check)
3. 58,27,12,x,2,1. Find x.
4. R-rounding off, M-modulus, T-truncate M(893,10)+r( )+t( ) is asked
5.vertices edges and surfaces of a cube

Ans.-8,12,6

6.Sums on Recursive functions


7.Questions on General computer awareness
Pick the odd one..
1.http 2.arp 3.snmp 4.sap Ans.-sap

Another..Ans.-Smtp

1.linux 2.windows NT 3.sql server 4.Unix Ans.-Sql server Freshersworld.com

Ans.-MVS

8. Which of the following is a singular matrix. (Determinant must be zero)


9. Aeroplane is flying at a particular angle and latitude,after some time another latitude is given..(8 hrs later), u r asked to
find the local time of the place.
10.a series of letters are given how many Ws r followed by F and preceded by T.
11. 7,9,13,_,27,37.

Ans.-19

12.SURFW Code is trAns.lated as SHEET..these kinda ques r there.. Freshersworld.com


13.194 base 10 = ____ base 5 (1234)
14.Largest prime no. in a 6 bit,8 bit

(Ans. 127),9 bit microprocessor

15.Venn Diagram kinda ques.


Some know English, some French,some Germanhow many know two languages..
16.Bar Diagram, Pie Chart (similar to Data interpretation)
17.Code Interchanging, A word is given. Letters are reversed..u r asked to find the nth letter from right or left.
Eg.
DESTABILIZATION

Ans.-T

18. Sums on logarithms, e power x curves. Freshersworld.com


19.n=68 x 12 x 51
Which of the follg is not an integer Ans.- n/122
20.Which is a/not a power of 2 or 3.

Power of 4 Ans.-4096

21. A-- 1 0 10 10 (Not exact values)


1. If VXUPLVH is written as SURMISE, what is SHDVD ?
Ans. PEASA (hint: in the first word, the alphabets of the jumbled one is three alphabets after the corresponding alphabet in
the word SURMISE. S = V-3, similarly find the one for SHDVD)
2. If DDMUQZM is coded as CENTRAL then RBDJK can be coded as --------Ans. QCEIL (hint: Write both the jumbled and the coded word as a table, find the relation between the corresponding words,
i.e C= D-1, N=M+1 & so on
3. In the word ECONOMETRICS, if the first and second , third and forth ,forth and fifth, fifth and sixth words are interchanged

Edited by: Riyaz Ali Khan Mayana

B. Tech .(CSE), D. E. C. E.

Email: riyaz666@gmail.com

38

up to the last letter, what would be the tenth letter from right?

Ans. word is CENOMOTEIRSC tenth

word is R
4. Find the result of the following expression if, M denotes modulus operation, R denotes round-off, T denotes truncation:
M(373,5)+R(3.4)+T(7.7)+R(5.8)

Ans. 19

5. What is the largest prime number that can be stored in an 8-bit memory?

Ans.

6. Find the physical quantity in units from the equation: (Force*Distance)/(Velocity*Velocity)

Ans. Ns2/m

7. Find the value of @@+25-++@16, where @ denotes "square" and + denotes "square root".

Ans: 621

8. If f(0)=1 and f(n)= f(n-1)*n, find the value of f(4).

Ans: 24

9. Convert the decimal number 310 to the base 6.

Ans: 1234

10. Find the missing number in the series: 2, 5, __ , 19 , 37, 75

Ans: 9

11. In a two-dimensional array, X(9,7), with each element occupying 4 bytes of memory, with the address of the first element
X(1,1) is 3000, find the address of X(8,5).

Ans.

12. Find the fourth row, having the bit pattern as an integer in an 8-bit computer, and express the Ans.wer in its decimal
value.
A00001111
B00110011
C01010101
(AU(B-C)) ?

Ans. 29

13. Complete the series 2, 7, 24, 77,__ (hint: 2*12= 24, 7*11= 77, therefore 24*10= 240)
14. Consider the following diagram for

Ans: 240

Ans.wering the following questions:

A. Find the difference between people playing cricket and tennis alone.

Ans: 4

B. Find the percentage of people playing hockey to that playing both hockey and cricket.

Ans:

C. Find the percentage of people playing all the games to the total number of players.

Ans: 6%

15. One more question of the same type (Same type of diagram; of course in a different set)
1. How many more or less speak English than French?
2. What % people speak all the three languages?
3. What % people speak German but not English?
{In another set cricket, hockey and tennis are changed with the name of some computer languages, such as Java, Cobol,
Fortran (may be some other name)}
16. Select the odd one out

a. Oracle b. Linux c. Ingress d. DB2

17. Select the odd one out

a. SMTP b. WAP c. SAP d. ARP

18. Select the odd man out.

a. Java b. Lisp c. Smalltalk d. Eiffel

19. Which of the following are orthogonal pairs?

a. 3i+2j b. i+j c. 2i-3j d. -7i+j

20. Number of faces, vertices and edges of a cube

a. 12,8,6 b. 4,6,8 c. 6,8,12 d. 6,12,8

21. Given a Bar Chart showing the sales of a company. (In Figure) The sales in years as shown in the figure are (in crores)
1998-1999 - 130, 1997-1998 - 90, 1996-1997 - 90, 1995-1996 - 70
1. The highest growth rate was for the year

Ans. 1998-1999

2. The net increase in sales of the company in the year span of 1995-1999

Ans. 60 crores.

3. The lowest growth rate was for the year

Ans. 1997

22. Find the value of the decimal number to the base 7.

Ans. 1436.

23. Complete the series:5,6,7,8,10,11,14,__.

Ans. 15

24. If the vertex (5,7) is placed in the memory. First vertex (1,1) s address is 1245 and then address of (5,7) is ---------25. In which of the system, decimal number 384 is equal to 1234?

Ans.

26. A man, a woman, and a child can do a piece of work in 6 days. Man only can do it in 24 days. Woman can do it in 16 days
and in how many days child can do the same work?

Ans.

27. In Madras, temperature at noon varies according to -t^2/2 + 8t + 3, where t is elapsed time. Find how much
Edited by: Riyaz Ali Khan Mayana

B. Tech .(CSE), D. E. C. E.

Email: riyaz666@gmail.com

39

temperature more or less in 4pm to 9pm.

Ans.

28. The size of the bucket is N kb. The bucket fills at the rate of 0.1 kb per millisecond. A programmer sends a program to
receiver. There it waits for 10 milliseconds. And response will be back to programmer in 20 milliseconds. How much time the
program takes to get a response back to the programmer, after it is sent?

Ans.

29. The size of a program is N. And the memory occupied by the program is given by M = square root of 100N. If the size of
the program is increased by 1% then how much memory now occupied ?

Ans.

30. A power unit is there by the bank of the river of 750 meters width. A cable is made from power unit to power a plant
opposite to that of the river and 1500mts away from the power unit. The cost of the cable below water is Rs. 15/- per meter
and cost of cable on the bank is Rs.12/- per meter. Find the total of laying the cable.
Ans. Rs. 22,500 (hint: the plant is on the other side of the plant i.e. it is not on the same side as the river)
{There are two questions, both showing a curve. In the first one, you have to identify the curve. In the second one you have
to Write the equation of the curve. In }
SECTION II 1. If A can copy 50 pages in 10 hours and A and B together can copy 70 pages in 10 hours, how much time does
B takes to copy 26 pages?
a. b. c. d.
2. Match the following:
1. Male - Boy ---> a. A type of
2. Square - Polygon ---> b. A part of
3. Roof - Building ---> c. Not a type of
4. Mushroom - Vegetables ---> d. A superset of
Ans.: 1- d, 2- a, 3- b, 4- c
3. Match the following.
1. brother - sister ---> a. Part of
2. Alsatian - dog ---> b. Sibling
3. sentence - paragraph ---> c. Type of
4. car - steering ---> d. Not a type of
Ans. 1-b, 2-c, 3-a, 4-d

1)In a well of 20feet depth, a frog jumps 5feet up in the morning and comes 4feet down in the evening,on which day the frog
gets out of the well. ____________________________________________________________________________
___________________ 2)Next number in the series: 77,49,36,18,? Ans.wer 8 (7*7=49)(4*9=36)(3*6=18) (1*8=8)
____________________________________________________________________________ ___________________ 3)A
& B are playing a game .There are 8 oranges in a row on the table.one Player can take 1-4 oranges in a pick (a maximum of
4),one who picks the last orange wins the game.'A' plays first How many oranges should he pick first time inorder to win the
game. Ans.wer 3. ____________________________________________________________________________
___________________ 4)9/10 or 10/11 which is greater.
____________________________________________________________________________ ____________________ 5)
(x-y/3)-(y-x/3)=? ____________________________________________________________________________
____________________ 6)x:y=3 and x+y=80 what is the value of y? Ans.wer y=20
____________________________________________________________________________ _____________________
7)Average of 5 number is -10 sum of 3 numbers is 16,what is the average of other two numbers? Ans.wer -33
____________________________________________________________________________ _____________________
8)16 litre can, 7 litre can,3 litre can,the customer has to be given 11 litres of milk using all the three cAns. only explain?
____________________________________________________________________________ ______________________
9)A car has run 10000 miles using 5 tyres interchangably,To have a equal wornout by all tyres how many miles each tyre
should have run. Ans.wer 4000 miles/tyre
____________________________________________________________________________ ______________________
10) 8 to the power of x is 32,what is the value of x?
____________________________________________________________________________ _____________________
Edited by: Riyaz Ali Khan Mayana

B. Tech .(CSE), D. E. C. E.

Email: riyaz666@gmail.com

40

11) 12 Blacksox and 12 Whitesox mixed in a box,a pair of sox is picked at a time,in which pick/ how many pick ,to get the
right pair(black&black or white&white)?
____________________________________________________________________________ _____________________
12)Two coins one with HEAD IN BOTH SIDES and the other coin HEAD IN ONE SIDE AND TAIL IN THE OTHER SIDE is in a
box,a coin is taken at random and FOUND HEAD IN ONE SIDE ..what is the probability that THE OTHER SIDE IS HEAD?
____________________________________________________________________________ _____________________
13)A man shapes 3 cardboards in 50 minutes,how many cardboards does he shape in 5 hours? Ans.wer 18cardboards.
____________________________________________________________________________ _____________________
14)How many 3 digits with 3 Distinct digits are there from 100-1000.
____________________________________________________________________________ _____________________
15)Three men goes to a hotel to stay ,the clerk says $30 per room/day so all the three plAns. to stay in one room so each
pays $10.After some time the clerk realises that he made a mistake of collecting $30 but the room cost only $25,there fore
he decides to return $5 to them so he calls the roomboy and gives him $5 asking him to return.The roomboy keeps $2 with
him and he returns only $3($1 for each). Now Totally all have paid $9 each($27)+rommboy $2 which is equal to $27.where
did $1 go,who has made THE MISTAKE

1. WHICH IS THE NEXT NO:


2. BFGE CODED AS CEHD

5,6,7,8,10,11,14,?
THEN CODE

PVHDJ

ANS.: 18
ANS.: QUICK

3. FIND THE NO. OF Y FOLLOWED BY W BUT THAT IS NOT FOLLOWED BY Z.

Y W R U D D Y W Z ...............

4. WHAT IS THE LARGEST PRIME NO THAT IS STORED IN 8 BIT PATTERN

ANS.: 253 (NOT SURE)

5.WHICH WILL GIVE GOOD STANDARD DEVIATION

2. (7,-7,7,-7,7)

6. WHICH IS NOT A SIDE OF A RECTANGULAR

1. (7,0,-7,0,7)
1. (2,3,4)

2.(3,4,7)

3. (1,0,-1,0,1)
3. (3,5,9)

7.WHICH SHAPE WILL BE OBTAINED BY USING THESE VALUES OF X ,Y


X

0.00001

10

1.02

100

1.72

1000

3.00

9999

4.72

8. WHICH EQUATION THAT BEST SUITS THIS CURVE


A LINE CUTS X AT -1 WHEN Y=0 AND X=0 WHEN Y=3 AND GOES UPWARD
Y
X
9. a man , woman and a boy joindly did a job in 6 days. a man alone finishes in 10 days, a women alone finish in 24 days.
then how many days the boy can take to finsh?
10. FOR TEMPERATURE A FUNCTION IS GIVEN ACCORDING TO TIME : ((t**2)/6) + 4t +12
WHAT IS THE TEMPERATURE RISE OR FALL BETWEEN 4.AM TO 9 AM
11. An aeroplane starts from a (some latitude is given according to place)at 2 am local time to b(some latitude). Travellling
time is 10 hours. What is the local time of b when it reaches b

Edited by: Riyaz Ali Khan Mayana

B. Tech .(CSE), D. E. C. E.

Email: riyaz666@gmail.com

41

12. A FILE IS TRANS.FERRED FROM A PLACE TO ADESTINATION CAPABLE OF 10 KB . THEY GIVEN SOME RATE OF
TRANS.FER. U HAVE FIND A EQUATION THAT BEST SUIT THIS.
13. IN A PLANAR CUBE , THE NO. OF VERTICES, NO OF EDGES AND NO OF FACES ARE
1. 6,6,6 2. 4,8,12

3....

4.........

14. VENN DIAGROM below


1.

HOW MANY PERSON KNOW ENGLISH MORE THAN FRENCH

2.

HOW MUCH % OF PEOPLE KNOWS ALL THE 3 LANGUAGES

3.

HOW MUCH % OF PEOPLE THOSE WHO KNOWS FRENCH AND GERMAN AND NOT ENGLISH

FRENCH
15. CORRECT CHART WITH CORRECT VALUES

140
120
100
80
YEAR

60
40
20
0

1995

1996

1997

1998

1999

X- YEAR
Y- NO OF PERSONS
1.

AVERAGE NO. OF PERSONS FROM 1995 - 1999

2.

WHICH YEAR HAS LARGE DIFFERENCES IN NO OF PERSONS

3.

IF 10% OF PEOPLE LEAVES THE OFFICE IN 1998 THEN, HOW MANY FRESH CANDIDATES CAN BE ADDED IN THE
NEXT YEAR

16. WHAT IS THE VALUE OF


M(373, 7) + R(6.8) -T(3.4) + R( 3.4)
M- MODULAS

R- ROUNDOFF

T- TRUNCATE

17. WHAT IS THE VALUE OF


% # % (5) + # % # (2)
WHERE

% - DOUBLING

#- RECIPROCAL

18. MATCH THE FOLLOWING


A

1. SENTENCE, PARAGRAPH

1. TYPE OF

2. BASMATI, WHEAT

2. A PART OF

3. BROTHER, SISTER

3. NOT A TYPE OF

4. BREIGAL, DOG

4. SIBLING

ANS.: 1-> 2

2->1

3->3

4->4

19. G (0) =1 G (1)= -1 IF G (N)=2* (G (N-1))- 3(G (N-2)) THEN WHAT IS THE VALUE
OF G (4)?
20.

(AUB)C
21.

TIME

DEGREE

Edited by: Riyaz Ali Khan Mayana

B. Tech .(CSE), D. E. C. E.

Email: riyaz666@gmail.com

42

7 6 43.15

5 31 4.3

THEN WHAT WILL BE THE DEGREE WHEN TIME IS 3 O CLOCK


22.

THREE COMPANIES WORKING INDEPENDENTLY AND GET SAVINGS 10%,


20%, 25%. IF THE COMPANIES WORK JUST OPPOSITE THEN WHAT WILL BE THE NET SAVING?

23. WHICH ONE WILL BE THE EXACT POWER OF 3


(i) 2768(ii)2678 (III) 2187
24. SOME RELATION THAT IS DEDUCE TO
A (POW 2) DIRECT PROPORTIONAL TO X (POW 3)
B (POW 2) DIRECT PROPORTIONAL TO Y (POW 3)
SOME FOUR ANS.WERS WERE GIVEN
ANS.: ALL OF THE ABOVE
25. 900 M WIDE 3000 M WIDTH
SOMETHING I CANT REMEMBER SOME VALUES ARE GIVEN
BY AIR PER M Rs. 4

BY GROUND PER M Rs. 5

THEN WHERE U WILL CUT


ODD MAN OUT
26. 1.JAVA

2.SMALLTALK

3.LISP

27. 1.SAP

2.ARP

3.WAP

4.EIFFEL
4.TCP IP

28. WHICH IS THE PERFECT ONE AMONG THE 4


1. 2x +3y=4

2. x + y= -1

3. Y=2x+ 3

C-Aptitude

1.What would be the output of the following program.


#include<stdio.h>
main()
{
extern int a;
printf("%d",a);;
}
int a=20;
(a) 20

(b) 0

(c) garbage value

(d) error!!

2.What would be the output of the following program.


main()
{
int a[5]={2,3};
printf("\n %d %d %d",a[2],a[3],a[4]);
}
(a) garbage value

(b) 2

3.What would be the output of the following program.


Edited by: Riyaz Ali Khan Mayana B. Tech .(CSE), D. E. C. E.

(c) 3

(d) 0

Email: riyaz666@gmail.com

43

main()
{
inti=-3,j=2,k=0,m;
m=++i&&++j||++k;
printf("\n %d %d %d %d",i,j,k,m);
}
(a) -2

(b) -3

(c) -2

(c) 3

15

(d) error

4.What would be the output of the following program.


main()
{
int a,b;
a=sumdig(123);
b=sumdig(123);
printf("%d %d",a,b);
}
sumdig(int n)
{
static int s=0;
int d;
if(n!=0)
{
d=n%10;
n=(n-d)/10;
s=s+d;
sumdig(n);
}
else

return(s);

}
(a) 12

(b) 6

12

(d) error

5.What would be the output of the following program.


#define CUBE(x) (x*x*x)
main()
{
int a,b=3;
a=CUBE(b++);
printf("\n %d %d",a,b);
}
(a) 64

(b) 27

(c) 27

(d) 64

6.What would be the output of the following program.


main()
{
const int x=get();
printf("%d",x);
}
get()
{
return(20);
Edited by: Riyaz Ali Khan Mayana

B. Tech .(CSE), D. E. C. E.

Email: riyaz666@gmail.com

44

}
(a) 20

(b) garbage value

7.A function has this prototype

(c) error

(d) 0

void f1(int **x),

How will you call this function?


(a) int **a;

(b) int a;

f1(a);

(c) int *a;

f1(&a);

(d) int a=5;

f1(&a);

f1(&&a);

8.pointout the error, if any, in the for loop


main()
{
int l=1;
for(;;)
{
printf("%d",l++);
if(l>10)
break;
}
}
(a) The condition in the for loop is a must
(c) The for loop should be replaced by awhile loop

(b) The two semicolons should be dropped


(d) No error

9.Can the following piece of code be executed?


int main(void)
{
char strA[10]="compile",strB[10];
my_strcpy(strB,strA);
puts(strB);
}
char * my_strcpy(char *destination,char *source)
{
char *p=destination;
while(*source!='\0')
{
*p++=*source++;
}
*p='\0';
return destination;
}
(a) Compilation will only give a warning but will proceed to execute & will display "compile"
(b) The compilation error char *(char *,char *) differs in levels of indirection from 'int()' will occur
(c) Yes & it will print compile on the screen

(d) None of the above

10.What would be the output of the following program.


#include<stdio.h>
main()
{
char str[5]="fast";
static char *ptr_to_array = str;
printf("%s",ptr_to_array);
Edited by: Riyaz Ali Khan Mayana

B. Tech .(CSE), D. E. C. E.

Email: riyaz666@gmail.com

45

}
(a) Compilation will only give a warning but will proceed to execute & will display "fast"
(b) display "fast" on screen

(c) will give a compilation error

(d) none of the above

11.What would be the output of the following program.


main()
{
int num,*p;
num=5;
p=&num;
printf("%d",*p);
}
(a) 6

(b) 5

(c) junk value

(d) compilation error

12.What would be the output of the following program.


main()
{
int a[3]={2,3,4};
char *p;
p=a;
p=(char *)((int *)p+1);
printf("%d",p);
}
(a) 2

(b) 0

(c) junk value

(d) 3

13.What would be the output of the following program.


main()
{
int i=10;
fn(i);
printf("%d",i);
}
fn(int i)
{
return ++i;
}
(a) 10

(b) 11

(c) 12

14. What will be the value of i & j after the loop isexecuted?<BR>
(a) i=4,j= 24

(b) i=24,j= 24

(c) i=25,j= 25

(d) Compilation error


for(i=0,j=0;i<5,j<25;i++,j++)
(d) i=5,j=25

15.What would be the output of the following program.


main()
{
int i,j;
i=10;
j=sizeof(++i);
printf("%d",i);
}
(a) 11

(b) 10

Edited by: Riyaz Ali Khan Mayana

B. Tech .(CSE), D. E. C. E.

(c) 4

(d) compilation error


Email: riyaz666@gmail.com

46

16.What would be the output of the following program.


main()
{
int i=7;
printf("%d\n",i++*i++);
}
(a) 49

(b) 56

(c) 72

(d) compilation error

17. What will the printf print?


main()
{
char *p,*f();
p=f();
printf("f() returns:%s\n",p);
}
char *f()
{
char result[80];
strcpy(result,"anything will do");
return (result);
}
(a) f() returns: anything will do
(c) compilation error

(b) f() returns:


(d) The printf statement is not going to be executed

18.How many times the following program would print 'Jamboree'?


main()
{
printf("\n Jamboree");
main();
}
(a) infinite number of times

(b) 32767 times

(c) 65535 times

(d) till the stack does not overflow

19.Notice the error in the default statement in the code snippet below.Will it give a compilation error?
main()
{
int a=10,j;
j=fn(a);
switch(j)
{
case 30: printf("the value is 30");
break;
case 50: printf("the value is 50");
break;
default:printf("the value is not 30 or 50");
}
}
fn(int a)
{
Edited by: Riyaz Ali Khan Mayana

B. Tech .(CSE), D. E. C. E.

Email: riyaz666@gmail.com

47

return (++a);
}
(a) Will display "the value is 30"

(b) Will display "The value is not 30 or 50"

(c) Yes a compilation error would happen


(d) No compilation errors but there will be no output on the screen
20.What would be the output of the following program.
main()
{
struct emp
{
char name[20];
int age;
float sal;
};
struct emp e = {"tiger"};
printf("\n %d %f",e.age,e.sal);
}
(a) 0

0.000000

(b) Garbage values

(c) Error

(d) none of the above

points regarding c:
1. the value of any variable will change as it is
executing
ex:
for(i=0,j=0;i<5,j<25;i++,j++)
%% the value of i and j will be 25 25 when it is printed

2. the value of any variable(i) will depend on the any values


given or specified
Ex:
main()
{
extern int a;
printf(a);
}
int a=5;

%% the value of a is 5

3. the value of any variable will not depend on the function


i.e scope

Edited by: Riyaz Ali Khan Mayana

B. Tech .(CSE), D. E. C. E.

Email: riyaz666@gmail.com

48

EX:
main()
{
int a;
pirntf(a);
}
scope()
{
int a=4;
}
%% the value of a is not 4 but a junk

4. when main is recursed the execution will continue untill


the stack gets overflow.

5. size of char * is 2
Answers are at last.
.L1.Q1 : Write the output of the following program
#include
#define

ABC

20

#define XYZ

10

#define XXX

ABC - XYZ

void main()
{
int

a;

a = XXX * 10;
printf("%d\n", a);
}
Solution for L1.Q1

L1.Q2 : Write the output of this program


#include
#define calc(a, b)

(a * b) / (a - b)

void main()
{
int a = 20, b = 10;
Edited by: Riyaz Ali Khan Mayana

B. Tech .(CSE), D. E. C. E.

Email: riyaz666@gmail.com

49

printf("%d\n", calc(a + 4, b -2));


}
Solution for L1.Q2

L1.Q3 : What will be output of the following program ?


#include
void main()
{
int cnt = 5, a;
do {
a /= cnt;
} while (cnt --);
printf ("%d\n", a);
}
Solution for L1.Q3

L1.Q4 : Print the output of this program


#include
void main()
{
int a, b, c, abc = 0;
a = b = c = 40;
if (c) {
int abc;
abc = a*b+c;
}
printf ("c = %d, abc = %d\n", c, abc);
}
Solution for L1.Q4

L1.Q5 : Print the output of this program


#include
main()
{
int k = 5;
if (++k < 5 && k++/5 || ++k <= 8);
Edited by: Riyaz Ali Khan Mayana

B. Tech .(CSE), D. E. C. E.

Email: riyaz666@gmail.com

50

printf("%d\n", k);
}
Solution for L1.Q5

L1.Q6 : What is the output of this program ?


#include
void fn(int, int);
main()
{
int

a = 5;

printf("Main : %d %d\n", a++, ++a);


fn(a, a++);
}
void fn(int a, int b)
{
printf("Fn : a = %d \t b = %d\n", a, b);
}
Solution for L1.Q6

Answers
L1.A1
Solution for L1.Q1
a = xxx * 10
which is => a = ABC - XYZ * 10
=> a = 20 - 10 * 10
=> a = 20 - 100
=> a = -80

L1.A2
Solution for L1.Q2
Actual substitution is like this :
calc(20+4, 10 -2) is calculated as follows
(20+4 * 10-2) / (20+4 - 10-2)
(20+40-2) / 12
58 / 12 = 4.8
since it is printed in %d the Ans. is 4

L1.A3
Solution for L1.Q3
Edited by: Riyaz Ali Khan Mayana

B. Tech .(CSE), D. E. C. E.

Email: riyaz666@gmail.com

51

This problem will compile properly, but it will give run


time error. It will give divide-by-zero error. Look in to
the do loop portion
do { a /= cnt; } while (cnt --);
when the 'cnt' value is 1, it is decremented in 'while
( cnt --)' and on next reference of 'cnt' it becomes zero.
a /= cnt; /* ie. a /= 0 */
which leads to divide-by-zero error.

L1.A4
Solution for L1.Q4
the result will be c = 40 and abc = 0;
because the scope of the variable 'abc' inside if(c) {.. }
is not valid out side that if (.) { .. }.

L1.A5
Solution for L1.Q5
The Ans.wer is 7. The first condition ++k < 5 is checked and
it is false (Now k = 6). So, it checks the 3rd condition
(or condition ++k <= 8) and (now k = 7) it is true. At this
point k value is incremented by twice, hence the value of k
becomes 7.

L1.A6
Solution for L1.Q6
The solution depends on the

implementation of stack.

(Depends on OS) In some machines the arguments are passed


from left to right to the stack. In this case the result
will be
Main : 5 7 Fn : 7 7
Other machines the arguments may be passed from right to
left to the stack. In that case the result will be
Main : 6 6
Fn

:87

1.void main()
{
int d=5;
printf("%f",d);
}
Edited by: Riyaz Ali Khan Mayana

B. Tech .(CSE), D. E. C. E.

Email: riyaz666@gmail.com

52

Ans.: Undefined

2.
void main()
{
int i;
for(i=1;i<4,i++)
switch(i)
case 1: printf("%d",i);break;
{
case 2:printf("%d",i);break;
case 3:printf("%d",i);break;
}
switch(i) case 4:printf("%d",i);
}
Ans.: 1,2,3,4

3.
void main()
{
char *s="\12345s\n";
printf("%d",sizeof(s));
}
Ans.: 6

4.
void main()
{
unsigned i=1; /* unsigned char k= -1 => k=255; */
signed j=-1; /* char k= -1 => k=65535 */
/* unsigned or signed int k= -1 =>k=65535 */
if(i<j)
printf("less");
else
if(i>j)
printf("greater");
else
if(i==j)
printf("equal");
}

Edited by: Riyaz Ali Khan Mayana

B. Tech .(CSE), D. E. C. E.

Email: riyaz666@gmail.com

53

Ans.: less

5.
void main()
{
float j;
j=1000*1000;
printf("%f",j);
}
1. 1000000
2. Overflow
3. Error
4. None
Ans.: 4
6. How do you declare an array of N pointers to functions returning
pointers to functions returning pointers to characters?
Ans.: The first part of this question can be Ans.wered in at least
three ways:
1. char *(*(*a[N])())();
2. Build the declaration up incrementally, using typedefs:
typedef char *pc;

/* pointer to char */

typedef pc fpc();

/* function returning pointer to char */

typedef fpc *pfpc;

/* pointer to above */

typedef pfpc fpfpc();


typedef fpfpc *pfpfpc;
pfpfpc a[N];

/* function returning... */
/* pointer to... */

/* array of... */

3. Use the cdecl program, which turns English into C and vice
versa:
cdecl> declare a as array of pointer to function returning
pointer to function returning pointer to char
char *(*(*a[])())()
cdecl can also explain complicated declarations, help with
casts, and indicate which set of parentheses the arguments
go in (for complicated function definitions, like the one
above).
Any good book on C should explain how to read these complicated
C declarations "inside out" to understand them ("declaration
mimics use").
The pointer-to-function declarations in the examples above have
Edited by: Riyaz Ali Khan Mayana

B. Tech .(CSE), D. E. C. E.

Email: riyaz666@gmail.com

54

not included parameter type information. When the parameters


have complicated types, declarations can *really* get messy.
(Modern versions of cdecl can help here, too.)

7. A structure pointer is defined of the type time . With 3 fields min,sec hours having pointers to intergers.
Write the way to initialize the 2nd element to 10.

8. In the above question an array of pointers is declared.


Write the statement to initialize the 3rd element of the 2 element to 10;
9.
int f()
void main()
{
f(1);
f(1,2);
f(1,2,3);
}
f(int i,int j,int k)
{
printf("%d %d %d",i,j,k);
}
What are the number of syntax errors in the above?
Ans.: None.
10.
void main()
{
int i=7;
printf("%d",i++*i++);
}
Ans.: 56
11.
#define one 0
#ifdef one
printf("one is defined ");
#ifndef one
printf("one is not defined ");
Ans.: "one is defined"
12.
void main()
{
Edited by: Riyaz Ali Khan Mayana

B. Tech .(CSE), D. E. C. E.

Email: riyaz666@gmail.com

55

intcount=10,*temp,sum=0;
temp=&count;
*temp=20;
temp=&sum;
*temp=count;
printf("%d %d %d ",count,*temp,sum);
}
Ans.: 20 20 20

13. There was question in c working only on unix machine with pattern matching.
14. what is alloca()

Ans. : It allocates and frees memory after use/after getting out of scope

15.
main()
{
static i=3;
printf("%d",i--);
return i>0 ? main():0;
}

Ans.: 321

16.
char *foo()
{
char result[100]);
strcpy(result,"anything is good");
return(result);
}
void main()
{
char *j;
j=foo()
printf("%s",j);
}

Ans.: anything is good.

17.
void main()
{
char *s[]={ "dharma","hewlett-packard","siemens","ibm"};
char **p;
p=s;
printf("%s",++*p);
printf("%s",*p++);
printf("%s",++*p);
}
Ans.: "harma" (p->add(dharma) && (*p)->harma)
"harma" (after printing, p->add(hewlett-packard) &&(*p)->harma)
"ewlett-packard"

Edited by: Riyaz Ali Khan Mayana

B. Tech .(CSE), D. E. C. E.

Email: riyaz666@gmail.com

56

1. What does the following program print?


#include <stio.h>
int sum,count;
void main(void)
{< BR>

for(count=5;sum+=--count;)

printf("%d",sum);
}
a. The pgm goes to an infinite loop

b. Prints 4791010974

d. Prints 5802112085

c. Prints 4791001974

e. Not sure

2. What is the output of the following program?


#include <stdio.h>
void main(void)
{
int i;< BR>

for(i=2;i<=7;i++)

printf("%5d",fno());
}
fno()
{
staticintf1=1,f2=1,f3;
return(f3=f1+f2,f1=f2,f2=f3);
}
a. produce syntax errors

b. 2 3 5 8 13 21 will be displayed

d. none of the above

e. Not sure

c. 2 2 2 2 2 2 will be displayed

3. What is the output of the following program?


#include <stdio.h>
void main (void)
{
int x = 0x1234;
int y = 0x5678;
x = x & 0x5678;
y = y | 0x1234;
x = x^y;
printf("%x\t",x);
x = x | 0x5678;
y = y & 0x1234;
y = y^x;
printf("%x\t",y);
}
a. bbb3

bbb7

b. bbb7

bbb3

d. 4448

444c

e. Not sure

c. 444c

4448

4. What does the following program print?


#include <stdio.h>
void main (void)
{
int x;
x = 0;
if (x=0)
Edited by: Riyaz Ali Khan Mayana

B. Tech .(CSE), D. E. C. E.

Email: riyaz666@gmail.com

57

printf ("Value of x is 0");


else
printf ("Value of x is not 0");
}
a. print value of x is 0

b. print value of x is not 0

d. there is a syntax error in the if statement

c. does not print anything on the screen


e. Not sure

5. What is the output of the following program?


#include <stdio.h>
#include <string.h>
int foo(char *);
void main (void)
{
char arr[100] = {"Welcome to Mistral"};
foo (arr);
}
foo (char *x)
{
printf ("%d\t",strlen (x));
printf ("%d\t",sizeof(x));
return0;
}
a. 100 100

b. 18 100

c. 18 18

d. 18 2

e. Not sure

6. What is the output of the following program?


#include <stdio.h>
display()
{
printf ("\n Hello World");
return 0;
}
void main (void)
{
int (* func_ptr) ();
func_ptr = display;
printf ("\n %u",func_ptr);
(* func_ptr) ();
}
a. it prints the address of the function display and prints Hello World on the screen
b. it prints Hello World two times on the screen
c. it prints only the address of the fuction display on the screen
d. there is an error in the program

e. Not sure

7. What is the output of the following program?


#include <stdio.h>
void main (void)
{
int i = 0;
char ch = 'A';
do
Edited by: Riyaz Ali Khan Mayana

B. Tech .(CSE), D. E. C. E.

Email: riyaz666@gmail.com

58

putchar (ch);
while(i++ < 5 || ++ch <= 'F');
}
a. ABCDEF will be displayed

b. AAAAAABCDEF will displayed

c. character 'A' will be displayed infinitely

d. none

e. Not sure

8. What is the output of the following program?


#include <stdio.h>
#define sum (a,b,c) a+b+c
#define avg (a,b,c) sum(a,b,c)/3
#define geq (a,b,c) avg(a,b,c) >= 60
#define lee (a,b,c) avg(a,b,c) <= 60
#define des (a,b,c,d) (d==1?geq(a,b,c):lee(a,b,c))
void main (void)
{
int num = 70;
char ch = '0';
float f = 2.0;
if des(num,ch,f,0) puts ("lee..");
else puts("geq...");
}
a. syntax error

b. geq... will be displayed

d. none

e. Not sure

c. lee.. will be displayed

9. Which of the following statement is correct?


a. sizeof('*') is equal to sizeof(int)
c. sizeof('*') is equal to sizeof(double)

b. sizeof('*') is equal to sizeof(char)


d. none

e. Not sure

10. What does the following program print?


#include <stdio.h>
char *rev(int val);
void main(void)
{
extern char dec[];
printf ("%c", *rev);
}
char *rev (int val)
{
char dec[]="abcde";
return dec;
}
a. prints abcde

b. prints the address of the array dec

c. prints garbage, address of the local variable should not returned

d. print a

e. Not sure

11. What does the following program print?


void main(void)
{
int i;
static int k;
if(k=='0')
Edited by: Riyaz Ali Khan Mayana

B. Tech .(CSE), D. E. C. E.

Email: riyaz666@gmail.com

59

printf("one");
else if(k== 48)
printf("two");
else
printf("three");
}
a. prints one

b. prints two

d. prints one three

e. Not sure

c. prints three

12. What does the following program print?


#include<stdio.h>
void main(void)
{
enum sub
{
chemistry, maths, physics
};
struct result
{
char name[30];
enum sub sc;
};
struct result my_res;
strcpy (my_res.name,"Patrick");
my_res.sc=physics;
printf("name: %s\n",my_res.name);
printf("pass in subject: %d\n",my_res.sc);
}
a. name: Patrick

b. name: Patrick

pass in subject: 2

pass in subject:3

d. gives compilation errors

c. name: Patrick
pass in subject:0

e. Not sure

13. What does


printf("%s",_FILE_); and printf("%d",_LINE_); do?
a. the first printf prints the name of the file and the second printf prints the line no: of the second printf in the file
b. _FILE_ and _LINE_ are not valid parameters to printf function
c. linker errors will be generated

d. compiler errors will be generated

e. Not sure

14. What is the output of the following program?


#include <stdio.h>
void swap (int x, int y, int t)
{
t = x;
x = y;
y = t;
printf ("x inside swap: %d\t y inside swap : %d\n",x,y);
}
void main(void)
{
int x;
Edited by: Riyaz Ali Khan Mayana

B. Tech .(CSE), D. E. C. E.

Email: riyaz666@gmail.com

60

int y;
int t;
x = 99;
y = 100;
swap (x,y,t);
printf ("x inside main:%d\t y inside main: %d",x,y);
}
a. x inside swap : 100

y inside swap : 99

x inside main : 100

y inside main : 99

b. x inside swap : 100

y inside swap : 99

x inside main : 99

y inside main : 100

c. x inside swap : 99

y inside swap : 100

x inside main : 99

d. x inside swap : 99

y inside swap : 100

x inside main : 100

y inside main : 100


y inside main : 99

e. Not sure
15. Consider the following statements:
i) " while loop " is top tested loop
iii) " do - while loop" is top tested loop

ii) " for loop " is bottom tested loop


iv) " while loop" and "do - while loop " are top tested loops.

Which among the above statements are false?


a. i only

b. i & ii

c. iii & i

d. ii, iii & iv

e. Not sure

16. Consider the following piece of code:


char *p = "MISTRAL";
printf ("%c\t", *(++p));
p -=1;
printf ("%c\t", *(p++));
Now, what does the two printf's display?
a. M M

b. M I

c. I M

d. M S

e. Not sure

17. What does the following program print?


#include <stdio.h>
struct my_struct
{
int p:1;
int q:1;
int r:6;
int s:2;
};
struct my_struct bigstruct;
struct my_struct1
{
char m:1;
};
struct my_struct1 small struct;
void main (void)
{
printf ("%d %d\n",sizeof (bigstruct),sizeof (smallstruct));
}
a. 10 1

b. 2 2

c. 2 1

d. 1 1

e. Not sure

18. Consider the following piece of code:


Edited by: Riyaz Ali Khan Mayana

B. Tech .(CSE), D. E. C. E.

Email: riyaz666@gmail.com

61

FILE *fp;
fp = fopen("myfile.dat","r");
Now fp points to
a. the first character in the file.
b. a structure which contains a char pointer which points to the first character in the file.
c. the name of the file.

d. none of the above.

e. Not sure.

19. What does the following program print?


#include <stdio.h>
#define SQR (x) (x*x)
void main(void)
{
int a,b=3;
a = SQR (b+2);
}
a. 25

b. 11

c. 17

d. 21

e. Not sure.

20. What does the declaration do?


int (*mist) (void *, void *);
a. declares mist as a function that takes two void * arguments and returns a pointer to an int.
b. declares mist as a pointer to a function that has two void * arguments and returns an int.
c. declares mist as a function that takes two void * arguments and returns an int.
d. there is a syntax error in the declaration.

e. Not sure.

21. What does the following program print?


#include <stdio.h>
void main (void)
{
int mat [5][5],i,j;
int *p;
p = & mat [0][0];
for (i=0;i<5;i++)
for (j=0;j<5;j++)
mat[i][j] = i+j;
printf ("%d\t", sizeof(mat)); < BR>

i=4;j=5;

printf( "%d", *(p+i+j));


}
a. 25 9

b. 25 5

c. 50 9

d. 50 5

e. Not sure

22. What is the output of the following program?


#include <stdio.h>
void main (void)
{
short x = 0x3333;
short y = 0x4321;
long z = x;
z = z << 16;
z = z | y;
printf("%1x\t",z);
Edited by: Riyaz Ali Khan Mayana

B. Tech .(CSE), D. E. C. E.

Email: riyaz666@gmail.com

62

z = y;
z = z >> 16;
z = z | x;
printf("%1x\t",z);
z = x;
y = x && y;
z = y;
printf("%1x\t",z);
}
a. 43213333

3333

b. 33334321

d. 43213333

4321

4321

e. Not sure

4321

4321

c. 33334321

3333

23. What is the output of the following program?


#include <stdio.h>
void main (void)
{
char *p = "Bangalore";
#if 0
printf ("%s", p);
#endif
}
a. syntax error #if cannot be used inside main function

b. prints Bangalore on the screen

c. does not print anything on the screen


d. program gives an error "undefined symbol if"

e. Not sure

24. If x is declared as an integer, y is declared as float, consider the following expression:


y = *(float *)&x;
Which one of the following statments is true?
a. the program containing the expression produces compilation errors;
b. the program containing the expression produces runtime errors;
c. the program containing the expression compiles and runs without any errors;
d. none of the above

e. Not sure

25. What is the return type of calloc function?


a. int *

b. void *

d. int

e. Not sure

c. no return type: return type is void

^^^^^^^^^^^^^^^^^^^^^^^^^^^^^^^^^^^^^^^^^^^^^^^^^^^^^^^^^^^^^^^^^^^^^^^^^^^^^
^^^^^^^^^^^^^^^^^^^^^^
21. What is true about the following C functions?
(A) Need not return any value.

(B) Should always return an integer.

(C) Should always return a float.

(D) Should always return more than one value.

22. enum number { a=-1, b=4, c,d,e,} what is the value of e?


(A) 7

(B) 4

(C) 5

(D) 3

23. Which of the following about automatic variables within a function is correct?
(A) Its type must be declared before using the variable.

(B) They are local.

(C) They are not initialized to zero.

(D) They are global.

Edited by: Riyaz Ali Khan Mayana

B. Tech .(CSE), D. E. C. E.

Email: riyaz666@gmail.com

63

24. Consider the following program segment


int n, sum=5;
switch(n)
{
case 2:sum=sum-2;
case 3:sum*=5;
break;
default:sum=0;
}
if n=2, what is the value of the sum?
(A) 0

(B) 15

(C) 3

(D) None of these.

25. Which of the following is not an infinite loop?


(A) x=0;

(B) # define TRUE 0....

do{

While(TRUE){....}
/*x unaltered within the loop*/

(C) for(;;)

{....}

....}
While(x==0);

(D) While(1) {....}

26. Output of the following program is


main()
{
int i=0;
for(i=0;i<20;i++)
{
switch(i){
case 0:
i+=5;
case 1:
i+=2;
case 5:
i+=5;
default:
i+=4;
break;
}
}
}
(A) 5,9,13,17

(B) 12,17,22

(C) 16,21

(D) syntax error.

27. What does the following function print?


func(int i)
{
if(i%2) return 0;
else return 1;
}
main()
{
int i=3;
Edited by: Riyaz Ali Khan Mayana

B. Tech .(CSE), D. E. C. E.

Email: riyaz666@gmail.com

64

i=func(i);
i=func(i);
printf("%d",i);
}
(A) 3

(B) 1

(C) 0

(D) 2

28. What will be the result of the following program?


char*g()
{
static char x[1024];
return x;
}
main()
{
char*g1="First String";
strcpy(g(),g1);
g1=g();
strcpy(g1,"Second String");
printf("Ans.wer is:%s", g());
}
(A) Ans.wer is: First String
(C) Run time Error/Core Dump

(B) Ans.wer is: Second String


(D) None of these

29. Consider the following program


main()
{
int a[5]={1,3,6,7,0};
int *b;
b=&a[2];
}
The value of b[-1] is
(A) 1

(B) 3

(C) -6

(D) none

30. Given a piece of code


int x[10];
int *ab;
ab=x;
To access the 6th element of the array which of the following is incorrect?
(A) *(x+5)

(B) x[5]

(C) ab[5]

(D) *(*ab+5} .

-------------------___________________________________________________________________________

1) Find the output for the following C program


main()
{
char *p1="Name";
char *p2;
p2=(char *)malloc(20);
while(*p2++=*p1++);
Edited by: Riyaz Ali Khan Mayana

B. Tech .(CSE), D. E. C. E.

Email: riyaz666@gmail.com

65

printf("%s\n",p2);
}

Ans.. An empty string

2) Find the output for the following C program


main()
{
intx=20,y=35;
x = y++ + x++;
y = ++y + ++x;
printf("%d %d\n",x,y);
}

Ans.. 57 94

3) Find the output for the following C program


main()
{
int x=5;
printf("%d %d %d\n",x,x<<2,x>>2);
}

Ans.. 5 20 1

4) Find the output for the following C program


< P>#defineswap1(a,b)a=a+b;b=a-b;a=a-b;
main()
{
intx=5,y=10;
swap1(x,y);
printf("%d %d\n",x,y);
swap2(x,y);
printf("%d %d\n",x,y);
}
int swap2(int a,int b)
{
int temp;
temp=a;
b=a;
a=temp;
return;
}

Ans.. 10 5

5) Find the output for the following C program


main()
{
char *ptr = "Ramco Systems";
(*ptr)++;
printf("%s\n",ptr);
ptr++;
printf("%s\n",ptr);
}

Edited by: Riyaz Ali Khan Mayana

Ans.. Samco Systems

B. Tech .(CSE), D. E. C. E.

Email: riyaz666@gmail.com

66

6) Find the output for the following C program


#include<stdio.h>
main()
{
char s1[]="Ramco";
char s2[]="Systems";
s1=s2;
printf("%s",s1);
}

Ans.. Compilation error giving it cannot be an modifiable 'lvalue'

7) Find the output for the following C program


#include<stdio.h>
main()
{
char *p1;
char *p2;
p1=(char *) malloc(25);
p2=(char *) malloc(25);
strcpy(p1,"Ramco");
strcpy(p2,"Systems");
strcat(p1,p2);
printf("%s",p1);
}

Ans.. RamcoSystems

8) Find the output for the following C program given that


[1]. The following variable is available in file1.c
static int average_float;

Ans.. All the functions in the file1.c can access the variable

9) Find the output for the following C program


# define TRUE 0
some code
while(TRUE)
{
some code
}

Ans.. This won't go into the loop as TRUE is defined as 0

10) Find the output for the following C program


main()
{
int x=10;
x++;
change_value(x);
x++;
Modify_value();
printf("First output: %d\n",x);
}
x++;
change_value(x);
printf("Second Output : %d\n",x);
Modify_value(x);
Edited by: Riyaz Ali Khan Mayana

B. Tech .(CSE), D. E. C. E.

Email: riyaz666@gmail.com

67

printf("Third Output : %d\n",x);


}
Modify_value()
{
return (x+=10);
}
change_value()
{
return(x+=1);
}

Ans.. 12 1 1

11) Find the output for the following C program


main()
{
int x=10,y=15;
x=x++;
y=++y;
printf("%d %d\n",x,y);
}

Ans.. 11 16

12) Find the output for the following C program


main()
{
int a=0;
if(a=0) printf("Ramco Systems\n");
printf("Ramco Systems\n");
}

Ans.. Ony one time "Ramco Systems" will be printed

13) Find the output for the following C program


#include<stdio.h>
int SumElement(int *,int);
void main(void)
{
int x[10];
int i=10;
for(;i;)
{
i--;
*(x+i)=i;
}
printf("%d",SumElement(x,10));
}
int SumElement(int array[],int size)
{
int i=0;
float sum=0;
for(;i<size;i++)
sum+=array[i];
return sum;
}

Edited by: Riyaz Ali Khan Mayana

B. Tech .(CSE), D. E. C. E.

Email: riyaz666@gmail.com

68

Q14) Find the output for the following C program


#include<stdio.h>
void main(void);
int printf(const char*,...);
void main(void)
{
inti=100,j=10,k=20;
-- int sum;
float ave;
charmyformat[]="ave=%.2f";
sum=i+j+k;
ave=sum/3.0;
printf(myformat,ave);
}
Q15) Find the output for the following C program
#include<stdio.h>
void main(void);
{
int a[10];
printf("%d",((a+9) + (a+1)));
}
Q16) Find the output for the following C program
#include<stdio.h>
void main(void)
{
struct s{
int x;
float y;
}s1={25,45.00};
union u{
int x;
float y;
} u1;
u1=(union u)s1;
printf("%d and %f",u1.x,u1.y);
}
Q17) Find the output for the following C program
#include<stdio.h>
void main(void)
{
unsigned int c;
unsigned x=0x3;
scanf("%u",&c);
switch(c&x)
{
case 3: printf("Hello!\t");
case 2: printf("Welcome\t");
Edited by: Riyaz Ali Khan Mayana

B. Tech .(CSE), D. E. C. E.

Email: riyaz666@gmail.com

69

case 1: printf("To All\t");


default:printf("\n");
}
}

Q18) Find the output for the following C program


#include<stdio.h>
int fn(void);
void print(int,int(*)());
int i=10;
void main(void)
{
int i=20;
print(i,fn);
}
void print(int i,int (*fn1)())
{
printf("%d\n",(*fn1)());
}
int fn(void)
{
return(i-=5);
}
Q19) Find the output for the following C program
#include<stdio.h>
void main(void);
{
char numbers[5][6]={"Zero","One","Two","Three","Four"};
printf("%s is %c",&numbers[4][0],numbers[0][0]);
}
Q20) Find the output for the following C program
int bags[5]={20,5,20,3,20};
void main(void)
{
int pos=5,*next();
*next()=pos;
printf("%d %d %d",pos,*next(),bags[0]);
}
int *next()
{
int i;
for(i=0;i<5;i++)
if (bags[i]==20)
return(bags+i);
printf("Error!");
exit(0);
}
Edited by: Riyaz Ali Khan Mayana

B. Tech .(CSE), D. E. C. E.

Email: riyaz666@gmail.com

70

Q21) Find the output for the following C program


#include<stdio.h>
void main(void)
{
inty,z;<BR>intx=y=z=10;
int f=x;
float Ans.=0.0;
f *=x*y;
Ans.=x/3.0+y/3;
printf("%d %.2f",f,Ans.);
}
Q22) Find the output for the following C program
#include<stdio.h>
void main(void);
{
doubledbl=20.4530,d=4.5710,dblvar3;
double dbln(void);
dblvar3=dbln();
printf("%.2f\t%.2f\t%.2f\n",dbl,d,dblvar3);
}
double dbln(void)
{
double dblvar3;
dbl=dblvar3=4.5;
return(dbl+d+dblvar3);
}

Q23) Find the output for the following C program


#include<stdio.h>
static int i=5;
void main(void)
{
int sum=0;
do
{
sum+=(1/i);
}while(0<i--);
}
Q24) Find the output for the following C program
#include<stdio.h>
void main(void)
{
intoldvar=25,newvar=-25;
int swap(int,int);
swap(oldvar,newvar);
Edited by: Riyaz Ali Khan Mayana

B. Tech .(CSE), D. E. C. E.

Email: riyaz666@gmail.com

71

printf("Numbers are %d\t%d",newvar,oldvar);


}
int swap(int oldval,int newval)
{
int tempval=oldval;
oldval=newval;
newval=tempval;
}
Q25) Find the output for the following C program
#include<stdio.h>
void main(void);
{
inti=100,j=20;
i++=j;
i*=j;
printf("%d\t%d\n",i,j);
}
Q26) Find the output for the following C program
#include<stdio.h>
void main(void);
int newval(int);
void main(void)
{
int ia[]={12,24,45,0};
int i;
int sum=0;
for(i=0;ia[i];i++)
{
sum+=newval(ia[i]);
}
printf("Sum= %d",sum);
}
int newval(int x)
{
static int div=1;
return(x/div++);
}

Q27) Find the output for the following C program


#include<stdio.h>
void main(void);
{
int var1,var2,var3,minmax;
var1=5;
var2=5;
var3=6;
minmax=(var1>var2)?(var1>var3)?var1:var3:(var2>var3)?var2:var3;
Edited by: Riyaz Ali Khan Mayana

B. Tech .(CSE), D. E. C. E.

Email: riyaz666@gmail.com

72

printf("%d\n",minmax);

Q28) Find the output for the following C program


#include<stdio.h>
void main(void);
{
void pa(int *a,int n);
int arr[5]={5,4,3,2,1};
pa(arr,5);
}
void pa(int *a,int n)
{
int i;
for(i=0;i<n;i++)
printf("%d\n",*(a++)+i);
}
Q29) Find the output for the following C program
#include<stdio.h>
void main(void);
void print(void);
{
print();
}
void f1(void)
{
printf("\nf1():");
}

Q30) Find the output for the following C program


#include "6.c"
void print(void)
{
extern void f1(void);
f1();
}
static void f1(void)
{
printf("\n static f1().");
}

Q31) Find the output for the following C program


#include<stdio.h>
void main(void);
static int i=50;
int print(int i);
void main(void)
Edited by: Riyaz Ali Khan Mayana

B. Tech .(CSE), D. E. C. E.

Email: riyaz666@gmail.com

73

{
static int i=100;
while(print(i))
{
printf("%d\n",i);
i--;
}
}
int print(int x)
{
static int i=2;
return(i--);
}

Q32) Find the output for the following C program


#include<stdio.h>
void main(void);
typedef struct NType
{
int i;
char c;
long x;
} NewType;
void main(void)
{
NewType *c;
c=(NewType *)malloc(sizeof(NewType));
c->i=100;
c->c='C';
(*c).x=100L;
printf("(%d,%c,%4Ld)",c->i,c->c,c->x);
}

Q33) Find the output for the following C program


#include<stdio.h>
void main(void);
const int k=100;
void main(void)
{
int a[100];
int sum=0;
for(k=0;k<100;k++)
*(a+k)=k;
sum+=a[--k];
printf("%d",sum);
}

Edited by: Riyaz Ali Khan Mayana

B. Tech .(CSE), D. E. C. E.

Email: riyaz666@gmail.com

74

_________________________________________________________________________________
1. Point out error, if any, in the following program
main()
{
int i=1;
switch(i)
{
case 1:
printf("\nRadioactive cats have 18 half-lives");
break;
case 1*2+4:
printf("\nBottle for rent -inquire within");
break;
}
}

Ans.. No error. Constant expression like 1*2+4 are acceptable in cases of a switch.

2. Point out the error, if any, in the following program


main()
{
int a=10,b;
a>= 5 ? b=100 : b=200;
printf("\n%d",b);
}

Ans.. lvalue required in function main(). The second assignment should be written in

parenthesis as follows:

a>= 5 ? b=100 : (b=200);

3. In the following code, in which order the functions would be called?


a= f1(23,14)*f2(12/4)+f3();
a) f1, f2, f3

b) f3, f2, f1

c) The order may vary from compiler to compiler

d) None of the above

4. What would be the output of the following program?


main()
{
int i=4;
switch(i)
{
default:
printf("\n A mouse is an elephant built by the Japanese");
case 1:
printf(" Breeding rabbits is a hair raising experience");
break;
case 2:
printf("\n Friction is a drag");
break;
case 3:
printf("\n If practice make perfect, then nobody's perfect");
}
}
a) A mouse is an elephant built by the Japanese
Edited by: Riyaz Ali Khan Mayana

B. Tech .(CSE), D. E. C. E.

b) Breeding rabbits is a hare raising experience


Email: riyaz666@gmail.com

75

c) All of the above

Ans.:

d) None of the above

5. What is the output of the following program?


#define SQR(x) (x*x)
main()
{
int a,b=3;
a= SQR(b+2);
printf("%d",a);
}
a) 25

b) 11

c) error

d) garbage value

6. In which line of the following, an error would be reported?


1.

#define CIRCUM(R) (3.14*R*R);

2.

main()

3.

4.

float r=1.0,c;

5.

c= CIRCUM(r);

6.

printf("\n%f",c);

7.

if(CIRCUM(r))==6.28)

8.
9.

printf("\nGobbledygook");
}

a) line 1

b) line 5

c) line 6

d) line 7

7. What is the type of the variable b in the following declaration?


#define FLOATPTR float*
FLOATPTR a,b;
a) float

b) float pointer

c) int

d) int pointer

8. In the following code;


#include<stdio.h>
main()
{
FILE *fp;
fp= fopen("trial","r");
}
fp points to:
a) The first character in the file.
b) A structure which contains a "char" pointer which points to the first character in the file.
c) The name of the file.

d) None of the above.

9. We should not read after a write to a file without an intervening call to fflush(), fseek() or rewind() < TRUE/FALSE>
Ans.. True
10. If the program (myprog) is run from the command line as

myprog 1 2 3 ,

What would be the output?

main(int argc, char *argv[])


{
int i;
for(i=0;i<argc;i++)
printf("%s",argv[i]);
Edited by: Riyaz Ali Khan Mayana

B. Tech .(CSE), D. E. C. E.

Email: riyaz666@gmail.com

76

}
a) 1 2 3

b) C:\MYPROG.EXE 1 2 3

c) MYP

d) None of the above

11. If the following program (myprog) is run from the command line as

myprog 1 2 3,

What would be the output?

main(int argc, char *argv[])


{
int i,j=0;
for(i=0;i<argc;i++)
j=j+ atoi(argv[i]);
printf("%d",j);
}
a) 1 2 3

b) 6

c) error

d) "123"

12. If the following program (myprog) is run from the command line as myprog monday tuesday wednesday thursday,
What would be the output?
main(int argc, char *argv[])
{
while(--argc >0)
printf("%s",*++argv);
}
a) myprog monday tuesday wednesday thursday

b) monday tuesday wednesday thursday

c) myprog tuesday thursday

d) None of the above

13. In the following code, is p2 an integer or an integer pointer?


typedef int* ptr
ptr p1,p2;

Ans.. Integer pointer

14. Point out the error in the following program


main()
{
const int x;
x=128;
printf("%d",x);
}

Ans.. x should have been initialized where it is declared.

15. What would be the output of the following program?


main()
{
int y=128;
const int x=y;
printf("%d",x);
}
Ans.: a) 128

b) Garbage value

c) Error

d) 0

16. What is the difference between the following declarations?


const char *s;
char const *s;
Ans.. No difference

Edited by: Riyaz Ali Khan Mayana

B. Tech .(CSE), D. E. C. E.

Email: riyaz666@gmail.com

77

17. What is the difference between the following declarations?


const char *const s;

char const *const s;

Ans.. No difference
18. What would be the output of the following program?
main()
{
char near * near *ptr1;
char near * far *ptr2;
char near * huge *ptr3;
printf("%d %d %d",sizeof(ptr1),sizeof(ptr2),sizeof(ptr3));
}
a) 1 1 1

b) 1 2 4

c) 2 4 4

19. If the following program (myprog) is run from the command line as

d) 4 4 4

myprog friday tuesday sunday,

What would be the output?


main(int argc, char*argv[])
{
printf("%c",**++argv);
}
a) m

b) f

c) myprog

20. If the following program (myprog) is run from the command line as

d) friday

myprog friday tuesday sunday,

What would be the output?


main(int argc, char *argv[])
{
printf("%c",*++argv[1]);
}
a) r

b) f

c) m

21. If the following program (myprog) is run from the command line as

d) y

myprog friday tuesday sunday,

What would be the output?


main(int argc, char *argv[])
{
while(sizeofargv)
printf("%s",argv[--sizeofargv]);
}
a) myprog friday tuesday sunday
c) sunday tuesday friday myprog

b) myprog friday tuesday


d) sunday tuesday friday

22. Point out the error in the following program


main()
{
int a=10;
void f();
a=f();
printf("\n%d",a);
}
void f()
{
Edited by: Riyaz Ali Khan Mayana

B. Tech .(CSE), D. E. C. E.

Email: riyaz666@gmail.com

78

printf("\nHi");
}
Ans.. The program is trying to collect the value of a "void" function into an integer variable.
23. In the following program how would you print 50 using p?
main()
{
int a[]={10, 20, 30, 40, 50};
char *p;
p= (char*) a;
}
Ans.. printf("\n%d",*((int*)p+4));
24. Would the following program compile?
main()
{
int a=10,*j;
void *k;< BR>

j=k=&a;

j++;
k++;
printf("\n%u%u",j,k);
}
a) Yes

b) No, the format is incorrect

c) No, the arithmetic operation is not permitted on void pointers


d) No, the arithmetic operation is not permitted on pointers
25. According to ANS.I specifications which is the correct way of declaring main() when it receives command line arguments?
a) main(int argc, char *argv[])
c) main()

{int argc;

char *argv[]; }

b) main(argc,argv)

int argc;

char *argv[];

d) None of the above

26. What error would the following function give on compilation?


f(int a, int b)
{
int a;
a=20;
return a;
}
a) missing parenthesis in the return statement
c) redeclaration of a

b) The function should be declared as int f(int a, int b)


d) None of the above

27. Point out the error in the following program


main()
{
const char *fun();
*fun()='A';
}
const char *fun()
{
return "Hello";
}
Edited by: Riyaz Ali Khan Mayana

B. Tech .(CSE), D. E. C. E.

Email: riyaz666@gmail.com

79

Ans.. fun() returns to a "const char" pointer which cannot be modified


28. What would be the output of the following program?
main()
{
const int x=5;
int *ptrx;
ptrx=&x;
*ptrx=10;
printf("%d",x);
}
a) 5

b) 10

Ans.:c) Error

d) Garbage value

29. A switch statement cannot include


a) constants as arguments

b) constant expression as arguments

c) string as an argument

d) None of the above

30. How long the following program will run?


main()
{
printf("\nSonata Software");
main();
}
a) infinite loop b) until the stack overflows
) All of the above

d) None of the above

31. On combining the following statements, you will get


a) char *p= malloc(100)

char*p;

p=malloc(100);

b) p= (char*)malloc(100)

c) All of the above

d) None of the above

32. What is the output of the following program?


main()
{
int n=5;
printf("\nn=%*d",n,n);
}
a) n=5
c) n=5

Ans.: b)n=

d) error

__________________________________________________________________________________

1. The C language terminator is


(a) semicolon

(b) colon

(c) period

(d) exclamation mark

2. What is false about the following -- A compound statement is


(a) A set of simple statements
(c) Can be used in place of simple statement
Edited by: Riyaz Ali Khan Mayana

B. Tech .(CSE), D. E. C. E.

(b) Demarcated on either side by curly brackets


(d) A C function is not a compound statement.
Email: riyaz666@gmail.com

80

3. What is true about the following C Functions


(a) Need not return any value

(b) Should always return an integer

(c) Should always return a float

(d) Should always return more than one value

4. Main must be written as


(a) The first function in the program

(b) Second function in the program

(c) Last function in the program

(d) Any where in the program

5. Which of the following about automatic variables within a function is correct ?


(a) Its type must be declared before using the variable

(b) They are local

(c) They are not initialized to zero

(d) They are global

6. Write one statement equivalent to the following two statements:

x=sqr(a); return(x);

Choose from one of the alternatives


(a) return(sqr(a));

(b) printf("sqr(a)");

(c) return(a*a*a);

(d) printf("%d",sqr(a));

7. Which of the following about the C comments is incorrect ?


(a) Comments can go over multiple lines
(b) Comments can start any where in the line
(c) A line can contain comments with out any language statements
(d) Comments can occur within comments
8. What is the value of y in the following code?
x=7;
y=0;
if(x=6) y=7;
else y=1;
(a) 7

(b) 0

(c) 1

(d) 6

9. Read the function conv() given below


conv(int t)
{
int u;
u=5/9 * (t-32);
return(u);
}
What is returned
(a) 15

(b) 0

(c) 16.1

(d) 29

10. Which of the following represents true statement either x is in the range of 10 and 50 or y is zero
(a) x >= 10 && x <= 50 || y = = 0
(c) y!=10 && x>=50

(b) x<50
(d) None of these

11. Which of the following is not an infinite loop ?


(a) while(1)\{ ....}

(b) for(;;){...}

(c) x=0;

(d) # define TRUE 0

do{ /*x unaltered within the loop*/


.....}while(x = = 0);
Edited by: Riyaz Ali Khan Mayana

...
while(TRUE){ ....}

B. Tech .(CSE), D. E. C. E.

Email: riyaz666@gmail.com

81

12. What does the following function print?


func(int i)
{
if(i%2)return 0;
else return 1;
}
main()
{
int =3;
i=func(i);
i=func(i);
printf("%d",i);
}
(a) 3

(b) 1

(c) 0

(d) 2

13. How does the C compiler interpret the following two statements
p=p+x;
q=q+y;
(a) p= p+x;

(b)p=p+xq=q+y;

(c)p= p+xq;

q=q+y;

(d)p=p+x/q=q+y;

q=q+y;

For questions 14,15,16,17 use the following alternatives:


a.int

b.char

c.string

d.float

14. '9'
15. "1 e 02"
16. 10e05
17. 15
18. Read the folllowing code
# define MAX 100
# define MIN 100
....
....
if(x>MAX)
x=1;
else if(x<MIN)
x=-1;
x=50;
if the initial value of x=200,what is the value after executing this code?
(a) 200

(b) 1

(c) -1

(d) 50

19. A memory of 20 bytes is allocated to a string declared as char *s then the following two statements are executed:
s="Entrance"
l=strlen(s);
what is the value of l ?
(a)20
Edited by: Riyaz Ali Khan Mayana

(b)8
B. Tech .(CSE), D. E. C. E.

(c)9

(d)21
Email: riyaz666@gmail.com

82

20. Given the piece of code


int a[50];
int *pa;
pa=a;
To access the 6th element of the array which of the following is incorrect?
(a) *(a+5)

(b) a[5]

(c) pa[5]

(d) *(*pa + 5}

21. Consider the following structure:


struct num nam
{
int no;
char name[25];
}
struct num nam n1[]={{12,"Fred"},{15,"Martin"},{8,"Peter"},{11,Nicholas"}};
.....
.....
printf("%d%d",n1[2],no,(*(n1 + 2),no) + 1);
What does the above statement print?
(a) 8,9

(b) 9,9

(c) 8,8

(d) 8,unpredictable value

22. Identify the in correct expression


(a)a=b=3=4;

(b)a=b=c=d=0;

(c)float a=int b= 3.5;

(d)int a; floatb;a=b=3.5;

23. Regarding the scope of the varibles;identify the incorrect statement:


(a) automatic variables are automatically initialized to 0

(b) static variables are are automatically initialized to 0

(c) the address of a register variable is not accessible

(d) static variables cannot be initialized with any expression

24. cond 1?cond 2?cond 3?:exp 1:exp 2:exp 3:exp 4;


is equivalent to which of the following?
(a) if cond 1
exp 1;
else if cond 2
exp 2;
else if cond 3
exp 3;
else exp 4;
(b) if cond 1
if cond 2
if cond 3
exp 1;
else exp 2;
else exp 3;
else exp 4;
(c) if cond 1 && cond 2 && cond 3
exp 1 |exp 2|exp 3|exp 4;
Edited by: Riyaz Ali Khan Mayana

B. Tech .(CSE), D. E. C. E.

Email: riyaz666@gmail.com

83

(d) if cond 3
exp 1;
else if cond 2 exp 2;
else if cond 3 exp 3;
else exp 4;
25. The operator for exponentiation is
(a) **

(b) ^

(c) %

(d) not available

(c) a>>=b

(d) a**=b

26. Which of the following is invalid


(a) a+=b

(b) a*=b

27. What is y value of the code if input x=10


y=5;
if (x==10)
else if(x==9)
else y=8;
(a)9

(b)8

(c)6

(d)7

28. What does the following code do?


fn(int n, int p, int r)
{
static int a=p;
switch(n)
{
case 4:a+=a*r;
case 3:a+=a*r;
case 2:a+=a*r;
case 1:a+=a*r;
}
}
(a) computes simple interest for one year

(b) computes amount on compound interest for 1 to 4 years

(c) computes simple interest for four year

(d) computes compound interest for 1 year

29.
a=0;
while(a<5)
printf("%d\\n",a++);
How many times does the loop occurs?
(a) infinite

(b)5

(c)4

(d)6

30. How many times does the loop iterated ?


for(i=0;i=10;i+=2)
printf("Hi\\n");
(a)10

(b) 2

(c) 5

(d) None of these

31. What is incorrect among the following


A recursive function
(a) calls itself

(b) is equivalent to a loop

(c) has a termination condition


Edited by: Riyaz Ali Khan Mayana

B. Tech .(CSE), D. E. C. E.

(d) does not have a return value at all


Email: riyaz666@gmail.com

84

32. Which of the following go out of the loop if expn 2 becoming false
(a) while(expn 1)\{...if(expn 2)continue;}

(b) while(!expn 1)\{if(expn 2)continue;...}

(c) do{..if(expn 1)continue;..}while(expn 2);

(d) while(!expn 2)\{if(expn 1)continue;..\}

33. Consider the following program


main()
{
unsigned int i=10;
while(i>=0)
{
printf("%u",i)
i--;
}
}
How many times the loop will get executed
(a)10

(b)9

(c)11

(d) infinite

34.Pick out the odd one out


(a) malloc()

(b) calloc()

(c) free()

(d) realloc()

35.Consider the following program


main()
{
int a[5]={1,3,6,7,0};
int *b;
b=&a[2];
}
The value of b[-1] is
(a) 1
36.

(b) 3

(c) -6

(d) none

# define prod(a,b)=a*b
main()
{
int x=2;
int y=3;
printf("%d",prod(x+2,y-10));
}

the output of the program is


(a) 8

(b) 6

(c) 7

(d) None

37.Consider the following program segment


int n,sum=1;
switch(n)
{
case 2:sum=sum+2;
case 3:sum*=2;
break;
default:sum=0;
}
Edited by: Riyaz Ali Khan Mayana

B. Tech .(CSE), D. E. C. E.

Email: riyaz666@gmail.com

85

If n=2, what is the value of sum


(a) 0

(b) 6

(c) 3

(d) None of these

(b) 1&3

(c) 3 only

38. Identify the incorrect one


1.if(c=1)
2.if(c!=3)
3.if(a<b)then
4.if(c==1)
(a) 1 only

(d) All of the above

39. The format specified for hexa decimal is


(a) %d

(b) %o

(c) %x

(d) %u

40. Find the output of the following program


main()
{
int x=5, *p;
p=&x
printf("%d",++*p);
}
(a) 5

(b) 6

(c) 0

(d) none of these

41.Consider the following C code


main()
{
int i=3,x;
while(i>0)
{
x=func(i);
i--;
}
int func(int n)
{
static sum=0;
sum=sum+n;
return(sum);
}
}
The final value of x is
(a) 6

(b) 8

(c) 1

(d) 3

43. Int *a[5] refers to


(a) array of pointers

(b) pointer to an array

(c) pointer to a pointer

(d) none of these

44.Which of the following statements is incorrect


(a)

typedef struct new


{
int n1;
char n2;
} DATA;

Edited by: Riyaz Ali Khan Mayana

B. Tech .(CSE), D. E. C. E.

Email: riyaz666@gmail.com

(b)

86

typedef struct
{
int n3;
char *n4;
}ICE;

(c)

typedef union
{
int n5;
float n6;
} UDT;

(d)

#typedef union
{
int n7;
float n8;
} TUDAT;

__________________________________________________________________________________
Q1. typedef struct{
char *;
nodeptr next;
} * nodeptr ;
What does nodeptr stand for?
Q2.

What does. int *x[](); meAns. ?

Q3.

struct list{
int x;
struct list *next;
}*head;
the struct head.x =100
Is the above assignment to pointer is correct or wrong ?

Ans.. Wrong

Q4.What is the output of the following ?


int i;
i=1;
i=i+2*i++;
printf(%d,i);
Q5.

Ans.. 4

FILE *fp1,*fp2;
fp1=fopen("one","w")
fp2=fopen("one","w")
fputc('A',fp1)
fputc('B',fp2)
fclose(fp1)
fclose(fp2)}

a.error

b.

Edited by: Riyaz Ali Khan Mayana

c.
B. Tech .(CSE), D. E. C. E.

d.

Ans.. no error. But It will over writes on same file.


Email: riyaz666@gmail.com

87

What are the output(s) for the following ?


Q6.

#include<malloc.h>
char *f()
{char *s=malloc(8);
strcpy(s,"goodbye")}
main()
{
char *f();
printf("%c",*f()='A');
}

Q7.

#define MAN(x,y) (x)>(y)?(x):(y)


{
inti=10;j=5;k=0;
k= MAX(i++,++j)
printf(%d %d %d %d,i,j,k)
}

Q8.

Ans.. 10 5 0

a=10;b= 5;c=3;d=3;
if(a<b)&&(c=d++)
printf(%d %d %d %d a,b,c,d)
else printf("%d %d %d %d a,b,c,d);

Q9.

#include<stdarg.h>
show(int t,va_list ptr1)
{
int a,x,i;
a=va_arg(ptr1,int)
printf("\n %d",a)
}
display(char)
{
int x;
listptr;
va_star(otr,s);
n=va_arg(ptr,int);
show(x,ptr);
}
main()
{
display("hello",4,12,13,14,44);
}

Q10.

main()
{
printf("hello");
fork();
}

Edited by: Riyaz Ali Khan Mayana

B. Tech .(CSE), D. E. C. E.

Email: riyaz666@gmail.com

Q11.

88

main()
{
int i = 10;
printf(" %d %d %d \n", ++i, i++, ++i);
}

Q12.

Ans.:? 13 11 11

#include<stdio.h>
main()
{
int *p, *c, i;
i = 5;
p = (int*) (malloc(sizeof(i)));
printf("\n%d",*p);
*p = 10;
printf("\n%d %d",i,*p);
c = (int*) calloc(2);
printf("\n%d\n",*c);
}

Q13.

#define MAX(x,y) (x) >(y)?(x):(y)


main()
{
inti=10,j=5,k=0;
k= MAX(i++,++j);
printf("%d..%d..%d",i,j,k);
}

Q14.

#include <stdio.h>
main()
{
enum _tag{ left=10, right, front=100, back};
printf("left is %d, right is %d, front is %d, back is %d",left,right,front,back);
}

Q15.

main()
{
inta=10,b=20;<BR>

a>=5?b=100:b=200;

printf("%d\n",b);
}

Q16.

#define PRINT(int) printf("int = %d ",int)


main()
{< BR>

intx,y,z;<BR>

x=03;y=02;z=01;

PRINT(x^x);
z<<=3;PRINT(x);
y>>=3;PRINT(y);
Edited by: Riyaz Ali Khan Mayana

B. Tech .(CSE), D. E. C. E.

Email: riyaz666@gmail.com

89

}
Q17.

#include<stdio.h>
main()
{
char s[] = "Bouquets and Brickbats";
printf("\n%c, ",*(&s[2]));
printf("%s, ",s+5);
printf("\n%s",s);
printf("\n%c",*(s+2));
}

Q18.

main()
{
struct s1
{
char *str;
struct s1 *ptr;
};
static struct s1 arr[] = { {"Hyderabad",arr+1},
{"Bangalore",arr+2},
{"Delhi",arr}
};
struct s1 *p[3];
int i; < BR>

for(i=0;i<=2;i++)

p[i] = arr[i].ptr;
printf("%s\n",(*p)->str);
printf("%s\n",(++*p)->str);
printf("%s\n",((*p)++)->str);
}
Q19. .main()
{
char *p = "hello world!";
p[0] = 'H';
printf("%s",p);
}

Ans.: Hello world!

_________________________________________________________________________________

PHYSCOMETRY TEST
the logic here is the same question will be repeated many times in many forms.for all of them u have to say the same
Ans.wer.very easy one .dont try to be honest ,but pretend to be correct.like if u say that u r easy in making friendships,stick
to it through out the paper .
Direction:
In this section you will find different questions with the same meaning. In all such questions your Ans.wer has to be same.
for e.g.:
In being thrown by chance with a stranger, you wait for the person to introduce himself or herself.
(a) Yes (b) No (c) ?
It is difficult for you to chat about things in general with people.
(a) Yes (b) No (c) ?
Edited by: Riyaz Ali Khan Mayana

B. Tech .(CSE), D. E. C. E.

Email: riyaz666@gmail.com

90

These two questions have similar meanings. If you Ans.wer the first one 'NO' and the second one 'YES', i.e. if you differ in
your Ans.wers to similar questions you lose marks for every question with the above meaning.
The choices to these questions are:

(a) Yes.

(b) No. (c) ?

Psychometric Test
Y-1.

You start to work on a project with great deal of enthusiasm.

N-2.

You would rather plan an activity than take part in it.

Y-3.

You have more than once taken lead in organizing project or a group of some kind.

Y-4.

You like to entertain guests.

N-5.

Your interests change quickly from one thing to another.

N-6.

When you eat a meal with others, you are usually one of the last to finish.

N-7.

You believe in the idea that we should " eat,drink and be merry, for tomorrow we die."

N-8.

When you find that something you have bought is defective, you hesitate to demand an exchange or a refund.

Y-9.

You find it easy to find new acquaintances.

N-10. You are sometimes bubbling over with energy and sometimes very sluggish.
Y-11. You are happiest when you get involved in some projects that calls for rapid action.
N-12. Other people think of you as being very serious minded.
N-13. In being thrown by chance with a stranger, you wait for the person to introduce himself or herself.
Y-14. You like to take part in many social activities.
N-15. You sometimes feel "just miserable" for no good reason at all.
N-16. You are often so much " on the go" that sooner or later you may wear yourself out.
Y-17. You like parties you attend to be lively.
Y-18. If you hold an opinion that is radically different that expressed by a lecturer, you are likely to tell the person about it
either during or after the lecture.
N-19. It is difficult for you to chat about things in general with people.
N-20. You give little thought to your failures after they are passed.
N-21. You often wonder where others get all the excess energy they seem to have.
Y-22. You are inclined to stop to think things over before you act.
N-23. You avoid arguing over a price with a clerk or sales person.
Y-24. You would dislike very much to work alone in some alone place.
N-25. You often find it difficult to go to sleep at night because you keep thinking of what happened during the day.
N-26. You find yourself hurrying to get to places even when there is plenty of time.
Y-27. You like work that requires considerable attention to details.
N-28. You are satisfied to let some one else take the lead in group activities.
Y-29. You enjoy getting acquainted with people.
Y-30. It takes a lot to get you emotionally stirred up or excited.
N-31. You work more slowly and deliberately than most people of your sex and age.
Y-32. You are a carefree individual.
N-33. When people do not play fair you hesitate to say anything about it to them.
N-34. It bothers you to have people watch you at your work.
Y-35. You have usually been optimistic about your future.
N-36. You like to have plenty of time to stop and rest.
Y-37. You take life very seriously.
Y-38. You enjoy applying for a job in person.
Y-39. You would like to be a host or hostess for parties at club.
N-40. You often feel uncomfortable or uneasy.
Y-41. You are the kind of person who is "on the go" all the time.
Y-42. You often crave excitement.
N-43. The thought of making a speech frightens you.
Edited by: Riyaz Ali Khan Mayana

B. Tech .(CSE), D. E. C. E.

Email: riyaz666@gmail.com

91

Y-44. You find it easy to start conversation with strangers.


N-45. You often feel guilty without a very good reason for it.
Y-46. People think you are a very energetic person.
N-47. You sometimes make quick decisions that you later wish you hadn't made.
N-48. You find it difficult to ask people for money or other donations, even for a cause in which you are interested.
Y-49. You are so naturally friendly that people immediately feel at ease with you.
N-50. You daydream a great deal.
Y-51. You are quick in your actions.
N-52. You have a habit of starting things and then losing interest in them.
Y-53. When you were a child many of your playmates naturally expected you to be the leader.
N-54. You sometimes avoid social contacts for fear of doing or saying the wrong thing.
N-55. You have frequent ups and downs in mood, sometimes with and sometimes without apparent cause.
Y-56. You always seem to have plenty of vigour and vitality.
N-57. It is difficult for you to understand people who get very concerned about things.
Y-58. When a clerk in a store waits on others who come after you, you call his or her attention to the fact.
Y-59. You would be very unhappy if you were prevented from making numerous social contacts.
N-60. There are times when your future looks very dark.
N-61. You sometimes wish that people would slow down a bit and give you a chance to catch up.
N-62. Many of your friends think you take your work too seriously.
N-63. You hesitate to walk into a meeting when you know that everyone's eye will be upon you.
N-64. You limit your friendships mostly to members of your own sex.
Y-65. You almost always feel well and strong.
N-66. You seem to lack the drive necessary to get as much as other people do.
N-67. You make decisions on the spur of the moment.
N-68. You are rather good at bluffing when you find yourself in difficulty.
N-69. After being introduced to someone , you just cannot think of things to say to make good conversation.
N-70. You feel lonesome even when with other people.
Y-71. You are able to work for unusually long hours without feeling tired.
N-72. You often act on the first thought that comes into your head.
Y-73. At the scene of an accident, you take an active part in helping out.
N-74. You have difficulty in making new friends.
N-75. Your mood often changes from happiness to sadness or vice versa without knowing why.
N-76. You talk more slowly than most people.
N-77. You like to play practical jokes upon others.
Y-78. You take the lead in putting life into a dull party.
Y-79. You would like to belong to as many clubs and social organizations as possible.
N-80. There are times when your mind seems to work very slowly and other times when it works very rapidly.
N-81. You like to do things slowly and deliberately.
N-82. You are a happy-go-lucky individual.
N-83. When you are served stale or inferior food in a restaurant, you say nothing about it.
N-84. You would rather apply for a job by writing a letter than by going through with a personal interview.
N-85. You are often in low spirits.
Y-86. You are inclined to rush from one activity to another without pausing enough for rest.
N-87. You are so concerned about the future that you do not get as much fun out of the present as you might.
Y-88. When you are attracted to a person whom you have not met earlier you make an active attempt to get -

acquainted

even though it may be quite difficult.


N-89. You are inclined to limit your acquaintances to select few
N-90. you seldom give your past mistakes a second thought.
N-91. You are less energetic than many people you know.
Y-92. You often stop to analyzed your thoughts and feelings.
Edited by: Riyaz Ali Khan Mayana

B. Tech .(CSE), D. E. C. E.

Email: riyaz666@gmail.com

92

Y-93. You speak out in meetings to oppose those whom you feel sure are wrong.
N-94. You are so shy it bothers you.
N-95. You are sometimes bothered by having a useless thought come into your mind over and over.
N-96. You get things in hurry.
N-97. It is difficult for you to understand how some people can be so unconcerned about the future.
N-98. You lie to sell things (i.e. to act as a sales person)
Y-99. You are often "Life of the Party".
N-100. You find daydreaming very enjoyable.
N-101. At work or at play other people find it hard to keep up with the pace you set.
Y-102. You can listen to a lecture without feeling restless.
Y-103. You would rather work for a good boss than for yourself.
Y-104. You can express yourself more easily in speech than in writing.
Y-105. You keep in fairly uniform spirits.
Y-106. You dislike to be hurried in your work.
N-107. You sometimes find yourself "crossing bridges before you come to them".
N-108. You find it somewhat difficult to say "no" to a sales person who tries to sell you something you do not - really want.
N-109. There are only a few friends with whom you can relax and have a good time.
Y-110. You usually keep cheerful in spite of trouble.
N-111. People sometimes tell you to "slow down" or "take it easy".
N-112. You are one of those who drink or smoke more than they know they should.
Y-113. When you think you recognize people you see in a public place, you ask them whether you have met - them before.
N-114. You prefer to work alone.
N-115. Disappointment affect you so little that you seldom think about them twice.
N-116. You are slow and deliberate in movements.
N-117. You like wild enthusiasm, sometimes to a point bordering on rowdyism at a football or baseball game.
N-118. You feel self conscious in the presence of important people.
Y-119. People think of you as being a very social type of person.
N-120. You have often lost sleep over your worries.
Y-121. You can turn out a large amount of work in a short time.
Y-122. You keep at a task until it is done, even after nearly everyone else has given up.
N-123. You can think of a good excuse when you need one.
N-124. Other people say that it is difficult to get to know you well.
N-125. You daydreams are often about things that can never come true.
N-126. You often run upstairs taking two steps at a time.
Y-127. You seldom let your responsibility interfere with your having a good time.
Y-128. You like to take on important responsibilities such as organizing a new business.
N-129. You have hesitated to make or to accept "dates" because of shyness.
N-130. Your mood is very easily influenced by people around you.
Y-131. Others are often amazed by the amount of work you turn out.
N-132. You generally feel as though you haven't a care in the world.
N-133. You find it difficult to get rid of sales person whom you do not care to listen or give your time.
N-134. You are a listener rather than a talker in a social conversation.
Y-135. You almost always feel that life is very much worth living.
N-136. It irritates you to have to wait at a crossing for a long freight train to pass.
N-137. You usually say what you feel like saying at the moment.
Y-138. You like to speak in public.
Y-139. You like to be with people.
Y-140. You generally keep cool and think clearly in exciting situations.
Y-141. Other people regard you as a lively individual.
Y-142. When you get angry, if you let yourself go, you feel better.
Edited by: Riyaz Ali Khan Mayana

B. Tech .(CSE), D. E. C. E.

Email: riyaz666@gmail.com

93

Y-143. You seek to avoid all trouble with other people.


Y-144. People seem to enjoy being with you.
N-145. You sometimes feel listless and tired for no good reason.
N-146. It is hard to understand why many people are so slow and get so little done.
N-147. You are fond of betting on horse races and games, whether you can afford it or not.
Y-148. If someone you know has been spreading untrue and bad stories about you, you see the person as soon as possible
and have a talk about it.
N-149. Shyness keep you from being as popular as you should be.
Y-150. You are generally free from worry about possible misfortunes.

Edited by: Riyaz Ali Khan Mayana

B. Tech .(CSE), D. E. C. E.

Email: riyaz666@gmail.com

Вам также может понравиться